इंदौर शाखा: IAS और MPPSC फाउंडेशन बैच-शुरुआत क्रमशः 6 मई और 13 मई   अभी कॉल करें
ध्यान दें:

आर्थिक सर्वेक्षण


भारतीय अर्थव्यवस्था

आर्थिक सर्वेक्षण 2023 (MCQ)

  • 27 Apr 2023
  • 188 min read

प्रश्न 1. आर्थिक सर्वेक्षण 2022-23 के संबंध में निम्नलिखित कथनों पर विचार कीजिये:

  1. इसे आर्थिक मामलों के विभाग के बजट प्रभाग द्वारा तैयार किया जाता है।
  2. यह FRBM अधिनियम 2003 के अंतर्गत अनिवार्य विवरणों में से एक है।

नीचे दिये गए कूट का प्रयोग कर सही उत्तर चुनिये:

(A) केवल 1
(B) केवल 2
(C) 1 और 2 दोनों
(D) न तो 1 न ही 2

उत्तर: D

व्याख्या: 

  • भारत का आर्थिक सर्वेक्षण वित्त मंत्रालय द्वारा जारी एक वार्षिक दस्तावेज़ है। यह आमतौर पर केंद्रीय बजट से एक दिवस पूर्व संसद में पेश किया जाता है।
  • यह मुख्य आर्थिक सलाहकार के मार्गदर्शन में आर्थिक मामलों के विभाग (DEA) के अर्थशास्त्र प्रभाग द्वारा तैयार किया जाता है। अतः कथन 1 सही नहीं है।
  • FRBM अधिनियम (राजकोषीय उत्तरदायित्व और बजट प्रबंधन अधिनियम) 2003 के अंतर्गत अनिवार्य विवरण: (a) वृहत-आर्थिक रूपरेखा विवरण (b) राजकोषीय नीति कार्ययोजना विवरण (c) मध्यावधिक राजकोषीय नीति विवरणअतः कथन 2 सही नहीं है।

प्रश्न 2. आपातकालीन क्रेडिट लाइन गारंटी योजना के संबंध में निम्नलिखित कथनों पर विचार कीजिये:

  1. यह MSMEs, व्यावसायिक उद्यमों तथा MUDRA उधारकर्ताओं एवं व्यावसायिक उद्देश्य हेतु व्यक्तिगत ऋणों के लिये संपार्श्विक (Collateral) मुक्त अतिरिक्त ऋण प्रदाय योजना है।
  2. 1000 करोड़ रुपये तक के वार्षिक कारोबार वाले कर्जदार योजना के अंतर्गत पात्र हैं।

उपर्युक्त कथनों में से कौन सा/से सही है/हैं?

(A) केवल 1
(B) केवल 2
(C) 1 और 2 दोनों
(D) न तो 1  न ही 2

उत्तर: A

व्याख्या:

आपातकालीन क्रेडिट लाइन गारंटी योजना:

  • उद्देश्य: वित्त मंत्री द्वारा घोषित इस योजना का उद्देश्य सूक्ष्म, लघु और मध्यम उद्यमों (MSMEs), व्यावसायिक उद्यमों तथा मुद्रा योजना (MUDRA Yojana) के उधारकर्त्ताओं को पूरी तरह से गारंटी एवं संपार्श्विक (Collateral) मुक्त अतिरिक्त ऋण प्रदान करना हैअतः कथन 1 सही है।
    • इस योजना के तहत 100 प्रतिशत संपार्श्विक (Collateral) मुक्त ऋण की गारंटी राष्ट्रीय ऋण गारंटी ट्रस्टी कंपनी लिमिटेड (NCGTC) द्वारा प्रदान की जा रही है, जबकि बैंक और गैर-बैंकिंग वित्तीय कंपनियाँ (NBFCs) योजना के तहत ऋण प्रदान करती हैं।
      • पात्रता: योजना के अंतर्गत  29 फरवरी, 2020 तक 50 करोड़ तक के ऋण बकाया वाले एवं 250 करोड़ रुपये तक के वार्षिक कारोबार करने वाले उधारकर्ता पात्र हैं। अतः कथन 2 सही नहीं है।

प्रश्न 3. निम्नलिखित में से कौन-सा/से बॉण्ड यील्ड में वृद्धि का/के कारण हो सकता/सकते है/हैं?

  1. ब्याज दर में वृद्धि।
  2. बॉण्ड की कीमतों में गिरावट।

नीचे दिये गए कूट का प्रयोग कर सही उत्तर चुनिये:

(A) केवल 1
(B) केवल2
(C) 1 और 2 दोनों
(D) न तो 1 न ही 2

उत्तर: C

व्याख्या:

  • अधिकांश बॉण्ड एक निश्चित ब्याज दर का भुगतान करते हैं जो ब्याज दरों में गिरावट से निवेशकों के लिये अधिक आकर्षक हो जाते है।
  • इसके विपरीत यदि ब्याज दरें बढ़ती हैं, तो निवेशक अब बॉण्ड द्वारा भुगतान की जाने वाली कम निश्चित ब्याज दर को पसंद नहीं करेंगे, जिसके परिणामस्वरूप इसकी कीमत में गिरावट आती है।
  • इस प्रकार, बॉण्ड का ब्याज दरों से विपरीत संबंध होता है। जब ऋण की लागत बढ़ती है (जब ब्याज दरें बढ़ती हैं), तो बॉण्ड की कीमतें आमतौर पर गिरती हैं। इसके अलावा, बॉण्ड की कीमतों में कमी से प्रतिफल में वृद्धि होती है। अतः विकल्प C सही है।

प्रश्न 4.निम्नलिखित में से कौन-सा संकुचनकारी मौद्रिक नीति का प्रभाव हो सकता है?

  1. वस्तुओं के मूल्य स्तर में वृद्धि।
  2. उच्च बेरोज़गारी।
  3. आर्थिक विकास में गिरावट।
  4. बचत और निवेश को प्रोत्साहन।

उपर्युक्त कथनों में से कौन-सा/से सही है/हैं?

(A) केवल 1
(B) केवल 2, 3
(C) केवल 2, 3 और 4
(D) 1, 2, 3 और 4

उत्तर: C

व्याख्या:

  • संकुचनकारी मौद्रिक नीति के प्रभाव :
    • मुद्रास्फीति को नियंत्रित करता है: अर्थव्यवस्था की मुद्रा आपूर्ति संकुचन का मुख्य उद्देश्य मुद्रास्फीति को कम करना और वस्तुओं तथा सेवाओं की कीमतों में स्थिरता सुनिश्चित करना है।
    • बेरोज़गारी बढ़ाता है: कम खपत के परिणामस्वरूप वस्तुओं और सेवाओं की बिक्री में भी गिरावट आती है। इस प्रकार कंपनियाँ व्यय को कम करने के लिये कर्मचारियों की छंटनी करती हैं।
    • आर्थिक विकास को धीमा करता है: चूँकि सभी उपाय अर्थव्यवस्था में मुद्रा आपूर्ति को कम करने पर ध्यान केंद्रित करते हैं इसलिये वस्तुओं और सेवाओं की खपत तथा बिक्री में कमी आती है। इसके परिणामस्वरूप व्यावसायिक संस्थाओं की लाभप्रदता कम हो जाती है। साथ ही बैंकों के पास अपने ग्राहकों को ऋण देने के लिये पर्याप्त धन नहीं होता है।
    • विनिमय दरों में वृद्धि: यह व्यय लेखों में कमी और आयात में वृद्धि की ओर ले जाता है। परिणामस्वरूप विनिमय दरें बढ़ती हैं तथा भुगतान संतुलन में गिरावट आती है।
    • बचत और निवेश को प्रोत्साहन: उपभोक्ता वस्तुओं और सेवाओं की खरीद पर कम खर्च करते हैं।
  • अतः विकल्प C सही है।

प्रश्न 5. वित्त वर्ष 2023 में भारत की वृद्धि 6.5 से 7.0% के मध्य रहने की उम्मीद है। आर्थिक सर्वेक्षण के अनुसार वित्त वर्ष 2023 में निम्नलिखित में से कौन-से भारत के विकास के चालक रहे हैं?

  1. घरेलू खपत में तेज़ी।
  2. सार्वजनिक पूंजी व्यय पर जोर।
  3. यह 'आधार प्रभाव' के लाभ के कारण है।

नीचे दिये गए कूट का प्रयोग कर सही उत्तर चुनिये:

(A) केवल 1 और 2
(B) केवल 2 और 3
(C) केवल 1
(D) 1, 2 और 3

उत्तर: A

व्याख्या:

  • अधोमुखी समीक्षा के बावजूद, FY23 के लिये विकास का अनुमान लगभग सभी प्रमुख अर्थव्यवस्थाओं की तुलना में अधिक है और यहाँ तक कि महामारी से पूर्व के दशक में भारतीय अर्थव्यवस्था की औसत वृद्धि तुलनात्मक रूप से अधिक रही है।
  • IMF ने वर्ष 2022 के लिये भारत को विश्व की शीर्ष दो तेज़ी से बढ़ती हुई महत्त्वपूर्ण अर्थव्यवस्थाओं में से एक रूप में अनुमानित किया था। विपरीत वैश्विक परिस्थितियों और संकुचनकारी घरेलू मौद्रिक नीति के बावजूद अगर भारत अभी भी 6.5 से 7.0% के बीच वृद्धि की उम्मीद रखता है और वह भी आधार प्रभाव के लाभ के बिना तो यह भारत के अंतर्निहित आर्थिक लचीलेपन को  प्रतिबिंबित करता  है। अतः कथन 3 सही नहीं है।
  • खपत में तेज़ी को पेंट-अप डिमांड (अटकी हुई माँग) के निर्गमन होने से भी समर्थन मिला है यह एक ऐसी घटना है जो पुनः भारत के लिये अद्वितीय नहीं है, किंतु फिर भी राष्ट्रिय प्रयोज्य आय की खपत में वृद्धि से प्रभावित एक सीमित घटना का प्रदर्शन करती है। अतः कथन 1 सही है।
  • भारत सरकार द्वारा पिछले दो बजटों में पूंजीगत व्यय पर ज़ोर देना कोई अलग पहल नहीं थी, जिसका उद्देश्य केवल देश में बुनियादी ढाँचे की कमी को दूर करना था, बल्कि यह एक रणनीतिक कार्ययोजना का हिस्सा थी जिसका उद्देश्य गैर-रणनीतिक PSE(विनिवेश) एवं सार्वजनिक क्षेत्र की संपत्तियों की निष्क्रियता से विस्तृत हुए आर्थिक परिदृश्य के खालीपन को निजी निवेश के माध्यम से भरना था। अतः कथन 2 सही है।

प्रश्न 6. निम्नलिखित में से क्या अर्थव्यवस्था में प्रयुक्त हॉकिश स्टांस शब्द के बारे में सही है?

(A) यह केंद्रीय बैंक की ओर से व्यय और धन की आपूर्ति तथा ब्याज दरों में कटौती की इच्छा को इंगित करता है।
(B) यह इंगित करता है कि केंद्रीय बैंक की सर्वोच्च प्राथमिकता मुद्रास्फीति को कम रखना है।
(C) आम तौर केंद्रीय बैंक ऐसी नीति तब अपनाता है जब विकास को नीति समर्थन की आवश्यकता होती है और मुद्रास्फीति तत्काल चिंता का विषय नहीं होती है।
(D) यह नीति आमतौर पर तब अपनाई जाती है जब मुद्रास्फीति और विकास दोनों पर नीतिगत प्राथमिकता समान होती है।

उत्तर: B

व्याख्या:

  • यह एक आक्रामक नीति को इंगित करता है जिसके अंतर्गत केंद्रीय बैंक की सर्वोच्च प्राथमिकता मुद्रास्फीति को कम रखना है।
  • इसके दौरान, केंद्रीय बैंक मुद्रा आपूर्ति में गिरावट के लिये ब्याज दरों में वृद्धि कर माँग को कम करती है।
  • आक्रामक नीति संकुचित मौद्रिक नीति का भी संकेत देती है।
  • जब केंद्रीय बैंक दरें बढ़ाता है या मौद्रिक नीति को संकुचित करता है तो बैंक भी उधारकर्त्ताओं को हतोत्साहित करने के लिये ऋण पर अपनी ब्याज दर बढ़ाते हैं जिसके परिणामस्वरूप यह वित्तीय प्रणाली में माँग को कम करता है। अतः विकल्प B सही है।

प्रश्न 7. निम्नलिखित में से कौन-सा श्रम बल भागीदारी दर का सबसे अच्छा वर्णन करता है?

(A) कुल जनसंख्या में कामकाजी आयु वर्ग की जनसंख्या का प्रतिशत ।
(B) कामकाजी आयु वर्ग की जनसंख्या का प्रतिशत जो रोज़गार योग्य है।
(C) कामकाजी आयु वर्ग की जनसंख्या का प्रतिशत जो कार्यरत है या सक्रिय रूप से रोज़गार की तलाश कर रही है।
(D) कामकाजी आयु वर्ग की जनसंख्या का प्रतिशत जो औपचारिक क्षेत्र का हिस्सा है।

उत्तर: C

व्याख्या:

  • सेंटर फॉर मॉनिटरिंग इंडियन इकोनॉमी (CMIE) के अनुसार, श्रम बल में ऐसे लोग शामिल हैं जो 15 वर्ष या उससे अधिक आयु के हैं और निम्नलिखित दो श्रेणियों में से किसी एक से संबंधित हैं:
    • जो नियोजित हैं
    • जो बेरोज़गार हैं और कार्य करने के इच्छुक हैं तथा सक्रिय रूप से नौकरी की तलाश कर रहे हैं।
  • इन दो श्रेणियों में वह लोग शामिल हैं जो नौकरी के लिये माँग कर रहे हैं। LFPR इसी मांग को संदर्भित करता है। अतः विकल्प C सही है।

प्रश्न 8. वित्त वर्ष 2023 के लिये भारत के कर संग्रह के संदर्भ में निम्नलिखित कथनों पर विचार कीजिये:

  1. अप्रत्यक्ष कर संग्रह एक बार पुनः प्रत्यक्ष कर संग्रह से अधिक हो गया है।
  2. आमतौर पर अप्रत्यक्ष करों को प्रत्यक्ष करों की तुलना में प्रगतिशील माना जाता है।

उपर्युक्त कथनों में से कौन-सा/से सही है/हैं?

(A) केवल 1
(B) केवल 2
(C) 1 और 2 दोनों
(D) न तो 1 न ही 2

उत्तर: D

व्याख्या:

  • वित्तीय वर्ष 2023 के लिये भारतीय केंद्रीय बजट के अनुमानों के अनुसार  प्रत्यक्ष कर से राजस्व संग्रह में अत्यधिक वृद्धि रही है। भारत में कुल केंद्रीय कर संग्रह में प्रत्यक्ष करों की हिस्सेदारी 51.5% और अप्रत्यक्ष करों की हिस्सेदारी 48.5% हैअतः कथन 1 सही नहीं है।
  • प्रत्यक्ष कराधान के समर्थकों का तर्क है कि यह एक अधिक न्यायसंगत प्रणाली है, क्योंकि यह व्यक्तियों और निगमों पर करारोपण उनकी आय या संपत्ति के आधार पर करती है।जबकि अप्रत्यक्ष कर किसी भी आय स्तर के लिये बराबर होते हैं जो उन्हें प्रतिगामी बनाता है। अतः कथन 2 सही नहीं है।

प्रश्न 9. निम्नलिखित में से कौन-सा/से चालू खाता घाटा वृद्धि का कारण है/हैं?

  1. जीवाश्म ईंधन के आयात में वृद्धि
  2. FDI प्रवाह में वृद्धि
  3. निर्यात में कमी

नीचे दिये गए कूट का प्रयोग कर सही उत्तर चुनिये:

(A) केवल 1 और 2
(B) केवल 2 और 3
(C) केवल 1 और 3
(D) 1, 2 और 3

उत्तर: C

व्याख्या:

  • चालू खाता घाटे की स्थिति तब उत्पन्न होती है जब किसी देश द्वारा आयात की जाने वाली वस्तु एवं सेवाओं का कुल मूल्य उसके द्वारा निर्यात की जाने वाली वस्तु एवं सेवाओं के कुल मूल्य से अधिक हो जाता है। बढ़ते आयात और घटते निर्यात से चालू खाता घाटा में वृद्धि होती है अतः कथन 1 और 3 सही हैं।
  • वस्तुओं के निर्यात और आयात के संतुलन को व्यापार संतुलन कहा जाता है। व्यापार संतुलन 'चालू खाता संतुलन' का एक भाग है। वर्ष 2021 की एक पूर्व रिपोर्ट के अनुसार, उच्च तेल आयात, उच्च सोने का आयात CAD में वृद्धि करने वाले प्रमुख प्रेरक कारक हैं। जबकि FDI  प्रवाह निर्यात को बढ़ावा देता है इसलिये यह चालू खाता घाटे में योगदान नहीं करता है। अतः कथन 2 सही नहीं है।

प्रश्न 10. वर्ष 1991 में भारत में शुरू किये गए संरचनात्मक सुधारों का मुख्य उद्देश्य क्या था?

(A) तिरती विनिमय दर के माध्यम से प्रतिस्पर्धात्मकता बनाए रखना
(B) वास्तविक वृद्धि दर को न्यूनतम करना 
(C) निजी क्षेत्रक की फर्मों के प्रवेश को प्रतिबंधित करना 
(D) कई क्षेत्रकों में सार्वजनिक क्षेत्रक के एकाधिकार का समर्थन करना 

उत्तर : A

व्याख्या : 

संरचनात्मक सुधार, 1991:

  • व्यापार एवं निवेश को उदार बनाने के उद्देश्य से भुगतान में नुकसान के संतुलन को बनाए रखने के लिये वर्ष 1991 में संरचनात्मक सुधार किये गए।
  • सुधारों ने विनिमय दर को तिरती और प्रतिस्पर्धा को बनाए रखने हेतु आवश्यक रूप से मूल्यह्रास की अनुमति दी और कई क्षेत्रों में सार्वजनिक क्षेत्र के एकाधिकार को समाप्त करके निजी क्षेत्रक की फर्मों के प्रवेश को प्रोत्साहित किया।
  • संरचनात्मक सुधारों का अर्थव्यवस्था पर सकारात्मक प्रभाव पड़ा, वास्तविक वृद्धि दर वर्ष 1980 के दशक के दौरान औसतन 5.5% से बढ़कर वित्त वर्ष 1993 से वित्त वर्ष 2000 तक 6.3% हो गई और सकल घरेलू उत्पाद (GDP) में कुल वस्तु एवं सेवा व्यापार वर्ष 1990 में 17.2 % से बढ़ कर वर्ष 2000 में 30.6% तक हो गया। अतः विकल्प A सही है।

प्रश्न 11. आर्थिक सर्वेक्षण 2023 के अनुसार निम्नलिखित में से कौन-सा कथन भारत में प्रत्यक्ष विदेशी निवेश के रुझानों का वर्णन करता है?

  1. निजी क्षेत्र की न्यूनतम भागीदारी के कारण इसका क्षय हुआ है।
  2. वर्ष 1990 के सुधारों के बाद इसमें वृद्धि हुई; हालाँकि कुछ वर्षों के बाद इसमें क्षय देखने को मिला।
  3. दूरसंचार क्षेत्र में समग्र सुधार के बाद किसी भी वित्तीय वर्ष में इसमें बिना क्षय हुए तेज़ी से वृद्धि हुई है।
  4. वर्ष 2000-2003 में निवेशीय उदारीकरण के परिणामस्वरूप, इसमें तेज़ी से वृद्धि हुई है।

नीचे दिये गए कूट का प्रयोग कर सही उत्तर को चुनिये:

(A) केवल 1 और 3
(B) केवल 2 और 4
(C) केवल 1, 2 और 3
(D) 1, 2, 3 और 4

उत्तर : B

व्याख्या :

  • वर्ष 1990 के दशक में उत्पाद और पूँजी बाज़ार में सुधार धीरे-धीरे जारी रहा। दशक के अंत के करीब सरकार ने उन्हें नए रूप में प्रस्तुत किया।
  • गैर-ऋण उत्पन्न करने वाले पूँजी प्रवाह के मुख्य स्रोत के रूप में प्रत्यक्ष विदेशी निवेश को प्रोत्साहित करने हेतु निवेश को और उदार बनाया गया।

  • अतः विकल्प B सही है।

प्रश्न 12. वृद्धि एवं विकास को बढ़ावा देने हेतु विगत आठ वर्षों में सरकार द्वारा क्या दृष्टिकोण अपनाया गया ?

(A) उत्पाद एवं पूँजी बाज़ार पर ध्यान केंद्रित करना
(B) विश्वास आधारित शासन और सार्वजनिक वस्तु की रणनीति अपनाना
(C) विकास प्रक्रिया में विभिन्न हितधारकों के बीच साझेदारी पर ज़ोर देना
(D) विकास के लिये निजी क्षेत्रकों को सेवा प्रदान करना

उत्तर : C

व्याख्या : 

  • वर्ष 2014 से पूर्व जो सुधार किये गए थे वे मुख्य रूप से उत्पाद और पूँजी बाज़ार पर केंद्रित थे, जो वर्ष 2014 के बाद भी जारी रहे।
  • हालाँकि सरकार ने, विगत आठ वर्षों में इन सुधारों को एक नया आयाम प्रदान किया। इसमें जीवनयापन और व्यवसाय करने में आसानी तथा आर्थिक दक्षता में सुधार पर अंतर्निहित बल के साथ, अर्थव्यवस्था की संभावित वृद्धि को ऊपर उठाने हेतु सुधारों को बेहतर किया गया है।
  • सुधारों के पीछे विस्तृत सिद्धांत जैसे सार्वजनिक वस्तुओं का निर्माण करना, विश्वास-आधारित शासन को अपनाना, विकास हेतु निजी क्षेत्र के साथ सह-साझेदारी करना और कृषि उत्पादकता में सुधार करना शामिल है।
  • यह दृष्टिकोण सरकार की वृद्धि एवं विकास रणनीति में एक आदर्श बदलाव को दर्शाता है, जिसमें विकास प्रक्रिया में विभिन्न हितधारकों के बीच साझेदारी के निर्माण पर ज़ोर दिया गया है, जहाँ हर कोई विकास में भाग लेता है और इससे लाभान्वित होता है (सबका साथ, सबका विकास)।
  • अतः विकल्प C सही है।

प्रश्न 13. डिजिटलीकरण ने वर्ष 2014 से वर्ष 2019 के मध्य भारत में आर्थिक विकास को कैसे प्रभावित किया है?

(A) इसने समग्र आर्थिक विकास को कम कर दिया है।
(B) अर्थव्यवस्था धीमी होने के साथ साथ कमज़ोर हो गई है।
(C) अर्थव्यवस्था को प्रभावित करने वाले अन्य कारकों की तुलना में अर्थव्यवस्था में तीव्र वृद्धि दर दर्ज की गई है।
(D) प्रारंभ में आर्थिक विकास स्थिर था लेकिन बाद में इसमें वृद्धि देखने को मिली।

उत्तर : C

व्याख्या :

  • भारत की डिजिटल अर्थव्यवस्था:
    • भौतिक बुनियादी संरचना पर ज़ोर देने के अतिरिक्त, विगत कुछ वर्षों के दौरान सार्वजनिक डिजिटल बुनियादी संरचना के विकास पर सरकार का ज़ोर व्यक्तियों और व्यवसायों की आर्थिक क्षमता को बढ़ाने में एक गेम चेंजर साबित हुआ है।
    • हाल ही में RBI के मासिक पत्रकों में प्रकाशित एक लेख का अनुमान है कि भारत की मुख्य डिजिटल अर्थव्यवस्था में वर्ष 2014 से वर्ष 2019 के मध्य समग्र आर्थिक विकास की तुलना में 2.4 गुना वृद्धि हुई है।
    • गैर-डिजिटल क्षेत्रकों के साथ अपने मज़बूत अग्रदर्शी जुड़ाव के साथ, डिजिटलीकरण विभिन्न माध्यमों जैसे उच्च वित्तीय समावेशन, अधिक औपचारिकता, बढ़ी हुई क्षमता और बेहतर अवसरों के माध्यम से संभावित आर्थिक विकास को मज़बूत करता है।
    • अतः विकल्प C सही है।

प्रश्न 14. आधार रुपी डिजिटल पहचान का भारतीय अर्थव्यवस्था पर क्या प्रभाव पड़ा ?

(A) औपचारिक ऋण तक बेहतर पहुँच
(B) व्यापारिक लेनदेन में आसानी
(C) लघु एवं मध्यम उद्यमों हेतु सूक्ष्म उद्यमों का विकास
(D) उपर्युक्त सभी

उत्तर : D

व्याख्या :

  • अर्थव्यवस्था पर आधार का महत्त्व:
    • आधार रुपी डिजिटल पहचान के निर्माण ने भारतीय अर्थव्यवस्था को औपचारिक बनाने में योगदान दिया है।
    • इसने कई असंगठित श्रमिकों और फुटपाथ विक्रेताओं के लिये औपचारिक ऋण तक पहुँच में सुधार किया है, साथ ही वस्तु एवं सेवा कर तंत्र (GSTN) और ई-वे बिल प्रणाली जैसी प्रणालियों के माध्यम से सरलीकृत और औपचारिक रूप से व्यापारिक लेनदेन भी किया है।
    • उद्यम पोर्टल पर पंजीकृत उद्यमों के माध्यम से यह देखा गया है कि लघु एवं मध्यम उद्यमों के लिये सूक्ष्म उद्यमों की वृद्धि भी डिजिटल पहचान द्वारा लाई गई वृद्धि की औपचारिकता का परिणाम है। अतः विकल्प D सही है।

प्रश्न 15. कंपनी अधिनियम, 2013 के संदर्भ में निम्नलिखित कथनों पर विचार कीजिये:

  1. यह कंपनी के निगमन, उत्तरदायित्वों, निदेशकों और विघटन को नियंत्रित करता है।
  2. इस अधिनियम में 'वन-पर्सन कंपनी' नामक एक नया शब्द शामिल किया गया है।

उपर्युक्त कथनों में से कौन-सा/से सही है/हैं?

केवल 1
केवल 2
1 और 2 दोनों
न तो 1 न ही 2

उत्तर : C

व्याख्या : 

  • कंपनी अधिनियम, 2013:
    • यह कंपनी के निगमन, उत्तरदायित्वों, निदेशकों और विघटन को नियंत्रित करता है। अतः कथन 1 सही है।
    • इसे 29 अध्यायों में विभाजित किया गया है, जिसमें पूर्व कंपनी अधिनियम, 1956 की 658 धाराओं की तुलना में 470 धाराएँ एवं 7 अनुसूचियाँ शामिल हैं।
    • इस अधिनियम में 'वन-पर्सन कंपनी' नामक एक नया शब्द शामिल किया गया है। अतः कथन 2 सही है।
    • वर्ष 2020 में, भारत की संसद ने कंपनी (संशोधन) विधेयक, 2020 को कंपनी अधिनियम में संशोधन करने और विभिन्न मिश्रित दोषों को न्यूनतम करने के साथ-साथ देश में व्यापार करने में आसानी को बढ़ावा देने हेतु पारित किया।
      • इसमें कुछ दोषों के साथ-साथ अधिकार निर्गम के लिये समयसीमा में कमी, व्यावसायिक सामाजिक उत्तरदायित्व (CSR) अनुपालन आवश्यकताओं में छूट और राष्ट्रीय कंपनी कानून अपीलीय न्यायाधिकरण (NCLAT) में अलग सूचकांक का निर्माण भी प्रस्तावित परिवर्तनों में से एक है।

प्रश्न 16. भारत में राजस्व उछाल में वृद्धि के पीछे मुख्य कारण कौन-सा है?

(A) केंद्रीय प्रत्यक्ष कर बोर्ड (CBDT) तथा केंद्रीय अप्रत्यक्ष कर और सीमा शुल्क बोर्ड (CBIC) के मध्य स्वचालित एवं नियमित आधार पर आँकड़े साझा करने का निर्णय।
(B) डिजिटल प्रणालियों में विभिन्न संगत जाँच के जरिये कर चोरी में कमी।
(C) बेहतर आय विवरण की वजह से प्रत्यक्ष कर संग्रहण में बढ़ोतरी हुई है।
(D) पहचान रहित निर्धारण और याचिका प्रणाली की शुरुआत।

उत्तर : B

व्याख्या :

  • उच्च राजस्व उछाल के पीछे एक प्रमुख कारण कर प्रक्रियाओं को आसान बनाने, अनुपालन में वृद्धि करने और धोखाधड़ी का पता लगाने वाली प्रणालियों में सुधार हेतु प्रौद्योगिकी समर्थित कर प्रशासन सुधारों की शुरूआत करना है।
  • पहचान रहित निर्धारण और याचिका प्रणाली को अब करदाताओं और आयकर विभाग के बीच भौतिक संपर्क की आवश्यकता नहीं है। इसके अतिरिक्त, एकीकृत डिजिटल प्रणाली के साथ विभिन्न संगत जाँच के जरिये कर चोरी को कम करती हैं।
    • उदाहरण के रूप में, GST रिटर्न दाखिल क्रियाविधि के परिणामस्वरूप आय विवरण बेहतरीन होता है, जिससे प्रत्यक्ष कर संग्रहण में बढ़ोतरी होती है।
  • इस संबंध में, CBDT और CBIC के मध्य स्वचालित एवं नियमित आधार पर आँकड़े और सूचना साझा करने का निर्णय एक आशाजनक सुधार है, इसके परिणामस्वरूप कर प्रणाली में लाभ होगा।
    • ये सुधार भविष्य के आर्थिक विकास और अर्थव्यवस्था में भविष्य में संसाधन जुटाने के लिये शुभ संकेत देते हैं। अतः विकल्प B सही है।

प्रश्न 17. सहस्राब्दी के दूसरे दशक में क्रेडिट-टू-जीडीपी अनुपात के स्वयं के रुझानों के विचलन का कारण क्या था?

(A) निजी गैर-वित्तीय क्षेत्र में निवेश के अवसरों की कमी।
(B) बैंकिंग क्षेत्रकों में डूबते ऋणों की संख्या में वृद्धि।
(C) निजी गैर-वित्तीय क्षेत्रकों में ऋण की माँग में कमी।
(D) बैंकिंग क्षेत्रकों की ऋण आपूर्ति में सुधार।

उत्तर : B

व्याख्या :

  • सकल घरेलू उत्पाद के प्रतिशत के रूप में निजी गैर-वित्तीय क्षेत्रकों के ऋण, सहस्राब्दी के दूसरे दशक में अधिकांश समय के लिये अपने रुझानों से निरंतर कम थे, जिसका अर्थ है सकल घरेलू उत्पाद के अनुपात में ऋणात्मक अंतर।
  • वर्ष 2017 में यह अंतर 25% तक बढ़ गया। जो सकल NPA और ऋण वृद्धि के बीच सांख्यिकीय रूप से नकारात्मक सहसंबंध (-0.5) को दर्शाता है, जिसके कारण दूसरे दशक के दौरान तुलन पत्र में प्रभाव के कारण बैंकों की ऋण आपूर्ति गंभीर रूप से बाधित हुई।
    • यह क्रेडिट-टू-जीडीपी अनुपात के रुझानों के बड़े नकारात्मक विचलन की व्याख्या करता है। अतः विकल्प B सही है।

प्रश्न 18. सरकार और भारतीय रिज़र्व बैंक (RBI) ने वर्ष 2010 के दौरान वित्तीय क्षेत्रकों के तुलन पत्र पर पड़ने वाले प्रभावों से उबरने में सहायता करने के लिये कौन-से उपाय किये ?

(A) सार्वजनिक क्षेत्र के बैंकों (PSB) का पुनर्पूंजीकरण
(B) सरफेसी अधिनियम, 2002 में संशोधन
(C) दिवाला और शोधन अक्षमता संहिता का कार्यान्वयन
(D) उपर्युक्त सभी

उत्तर : D

व्याख्या :

  • जैसे-जैसे कंपनियों द्वारा किया गया निवेश डूबता गया, उनकी बैंक ऋण आपूर्ति की क्षमता क्षीण होती गई। इसलिये, बैंकों की गैर-निष्पादित परिसंपत्तियाँ बढ़ने लगीं।
    • विगत दशक की दूसरी छमाही में वित्तीय और गैर-वित्तीय क्षेत्रकों के तुलन पत्र में सुधार की लंबी अवधि हेतु यह गतिविधियों का समूह है।
  • सरकार और RBI ने वर्ष 2010 के दौरान वित्तीय क्षेत्रकों को तुलन पत्र पर पड़ने वाले प्रभावों से उबरने में मदद करने के लिये कई नीतिगत पहलें कीं।
  • इनमें से कुछ जैसे SARFAESI अधिनियम 2002 में संशोधन, दिवाला और शोधन अक्षमता संहिता (IBC) का कार्यान्वयन, 'संपत्ति गुणवत्ता समीक्षा' (AQR) की शुरुआत, त्वरित सुधारात्मक कार्रवाई (PCA) संरचना की शुरुआत, सार्वजनिक क्षेत्रकों के बैंकों (PSB) का पुनर्पूंजीकरण, अन्य के बीच PSB के विलय से बैंकों/कॉर्पोरेटों के तुलन पत्र को शुद्ध करने में सहायता मिली। अतः विकल्प D सही है।

प्रश्न 19. राष्ट्रीय लॉजिस्टिक्स नीति (NLP), 2022 के संदर्भ में निम्नलिखित कथनों पर विचार कीजिये:

  1. यह नीति प्रमुख क्षेत्रों जैसे पुनरभियांत्रीकरण प्रक्रिया, डिजिटलीकरण और बहुविध परिवहन पर केंद्रित है।
  2. वर्ष 2030 तक लॉजिस्टिक्स की लागत को कम करके वैश्विक इंडेक्स में शीर्ष पर पहुँचने के लिये लॉजिस्टिक्स की लागत को आधा करना होगा।

उपर्युक्त कथनों में से कौन-सा/से सही है/हैं?

(A) केवल 1
(B) केवल 2
(C) 1 और 2 दोनों
(D) न तो 1 न ही 2

उत्तर : C

व्याख्या : 

राष्ट्रीय लॉजिस्टिक्स नीति (NLP), 2022:

  • परिचय:
    • यह नीति प्रमुख क्षेत्रों जैसे पुनरभियांत्रीकरण प्रक्रिया, डिजिटलीकरण और बहुविध परिवहन पर केंद्रित है। अतः कथन 1 सही है।
    • यह एक महत्त्वपूर्ण कदम है क्योंकि उच्च लॉजिस्टिक्स लागत अंतर्राष्ट्रीय बाज़ार में घरेलू सामानों की प्रतिस्पर्द्धात्मकता को प्रभावित करती है।
    • इसमें एक राष्ट्रीय लॉजिस्टिक्स नीति की आवश्यकता महसूस की गई है क्योंकि भारत में अन्य विकसित अर्थव्यवस्थाओं की तुलना में लॉजिस्टिक्स लागत अधिक है।
  • लक्ष्य:
    • इस नीति का उद्देश्य लागतों में कटौती करना है, जो वर्तमान में सकल घरेलू उत्पाद (GDP) का लगभग 14-15 प्रतिशत है। जिसमें वर्ष 2030 तक लगभग 8 प्रतिशत तक की कमी लाना है। अतः कथन 2 सही है।
      • अमेरिका, दक्षिण कोरिया, सिंगापुर और कुछ यूरोपीय देशों में लॉजिस्टिक्स लागत GDP अनुपात से कम है।
    • विश्व की 5वीं सबसे बड़ी अर्थव्यवस्था होने के नाते भारत का लक्ष्य वर्ष 2030 तक लॉजिस्टिक्स परफॉर्मेंस इंडेक्स (LPI) में शीर्ष 10 में शामिल होना है। उसे दक्षिण कोरिया की विकास गति की बराबरी करनी होगी।
      • भारत वर्ष 2018 में LPI में 44वें स्थान पर था।
    • कुशल लॉजिस्टिक्स पारिस्थितिकी तंत्र को सक्षम करने के लिये डेटा-संचालित निर्णय समर्थन प्रणाली (Decision Support Systems-DSS) बनाना।
    • नीति का लक्ष्य यह सुनिश्चित करना है कि लॉजिस्टिक मुद्दों को कम-से-कम किया जाए, निर्यात कई गुना बढ़े और छोटे उद्योगों एवं उनमें काम करने वाले लोगों को अधिक लाभ मिले। 

प्रश्न 20. निम्नलिखित में से कौन-सा कारक भारत में उद्योगों के लिये बैंक ऋण की वृद्धि में योगदान देता है?

(A) आपातकालीन क्रेडिट लिंक्ड गारंटी योजना (ECLGS) की शुरुआत
(B) MSME द्वारा माँगे गए ऋण में वृद्धि
(C) बड़े उद्योगों द्वारा अस्थिर ऋण और इक्विटी बाज़ारों से पूँजी जुटाने में कमी
(D) उपरोक्त सभी।

उत्तर: D

व्याख्या:

  • सभी तीन कारक (A, B और C) भारत में उद्योगों के लिये बैंक ऋण में वृद्धि में योगदान दे रहे हैं।
  • आपातकालीन क्रेडिट लिंक्ड गारंटी योजना (ECLGS) की शुरुआत से सहायता प्राप्त सूक्ष्म लघु और माध्यम उद्योगों (MSME) के ऋण में उल्लेखनीय वृद्धि देखी गयी है। जबकि कुल ऋण में वृद्धि,   सूक्ष्म लघु और मध्यम उद्योगों (MSME) द्वारा माँगे गए ऋण में वृद्धि से प्रेरित है।  
  • इसके अतिरिक्त, बड़े उद्योगों ने अपनी पूंजी जुटाने में अस्थिर ऋण और इक्विटी बाज़ारों से कमी की है और अपने ऋण की गति को बढ़ाना शुरू कर दिया है, जिससे सम्पूर्ण उद्योग को ऋण में वृद्धि में योगदान मिला है। अतः विकल्प D सही है।

प्रश्न 21.भारतीय फार्मास्युटिकल्स उद्योग के संबंध में निम्नलिखित कथनों पर विचार कीजिये:

  1. मात्रा के आधार पर फार्मा उत्पादों के उत्पादन में भारत दुनिया में तीसरे स्थान पर है।
  2. भारत जेनेरिक दवाओं का सबसे बड़ा वैश्विक आपूर्तिकर्त्ता है जो मात्रा के आधार पर वैश्विक आपूर्ति में 20% की हिस्सेदारी रखता है।

उपर्युक्त कथनों में से कौन-सा/से सही है/हैं?

(A) केवल 1
(B) केवल 2
(C) 1 और 2 दोनों
(D) उपरोक्त में से कोई नहीं

उत्तर- C

व्याख्या:

  • भारतीय फार्मास्युटिकल्स उद्योग वैश्विक फार्मास्यूटिकल्स उद्योग में एक प्रमुख भूमिका निभाता है।
    • मात्रा के आधार पर फार्मा उत्पादों के उत्पादन में भारत दुनिया भर में तीसरे और मूल्य के आधार पर 14वें स्थान पर है। अतः कथन 1 सही है।
  • यह क्षेत्र वैश्विक स्तर पर जेनरिक दवाओं का सबसे बड़ा प्रदाता है, जो मात्रा के आधार पर वैश्विक आपूर्ति में 20% की हिस्सेदारी रखता है और 60 प्रतिशत बाज़ार हिस्सेदारी के साथ विश्व स्तर पर अग्रणी वैक्सीन निर्माता भी है। अतः कथन 2 सही है।

प्रश्न 22. निम्नलिखित में से कौन-सा भारत का प्रमुख उद्योग नहीं है?

(A) कोयला
(B) सीमेंट
(C) कपड़ा
(D) फार्मास्यूटिकल्स 

उत्तर: D

व्याख्या:

  • भारत के आठ प्रमुख उद्योग क्षेत्र हैं जो देश के औद्योगिक उत्पादन की नींव बनाते हैं।
    • इन उद्योगों में कोयला, कच्चा तेल, प्राकृतिक गैस, रिफाइनरी उत्पाद, उर्वरक, स्टील, सीमेंट और बिजली शामिल हैं।
  • ये देश के समग्र आर्थिक विकास, रोज़गार के अवसर उत्पन्न करने और देश की जीडीपी में योगदान देने के लिये महत्वपूर्ण हैं।
    • फार्मास्यूटिकल्स आठ प्रमुख उद्योगों का हिस्सा नहीं है।
  • अतः विकल्प D सही है।

प्रश्न 23.  समग्र सकल मूल्यवर्धन (GVA) क्या है?

(A) किसी अर्थव्यवस्था में उत्पादन प्रक्रिया में उपयोग किये जाने वाले मध्यवर्ती इनपुट के मूल्य को छोड़कर, उत्पादित वस्तुओं और सेवाओं के कुल मूल्य का माप।
(B) किसी अर्थव्यवस्था में उत्पादित वस्तुओं और सेवाओं के कुल मूल्य का एक माप, जिसमें उत्पादन प्रक्रिया में उपयोग किये जाने वाले मध्यवर्ती इनपुट का मूल्य भी शामिल है।
(C) एक अर्थव्यवस्था के निवासियों द्वारा अर्जित कुल आय का एक उपाय।
(D) एक अर्थव्यवस्था के निवासियों द्वारा किये गए कुल व्यय का एक उपाय।

उत्तर: A 

व्याख्या: 

  • समग्र सकल मूल्यवर्धन (GVA) कच्चे माल, उपयोगिताओं और सेवाओं जैसे उत्पादन प्रक्रिया में उपयोग किये जाने वाले मध्यवर्ती इनपुट के मूल्य को छोड़कर, अर्थव्यवस्था में उत्पादित वस्तुओं और सेवाओं के कुल मूल्य का एक उपाय है।
    • यह अर्थव्यवस्था के विभिन्न क्षेत्रों द्वारा जोड़े गए मूल्य का माप है और इसका उपयोग किसी देश के आर्थिक प्रदर्शन का आकलन करने के लिये किया जाता है।
  • किसी देश के GVA का उपयोग उन क्षेत्रों की पहचान करने के लिये किया जा सकता है जो आर्थिक विकास को गति दे रहा है तथा देश के समग्र आर्थिक विकास में विभिन्न क्षेत्रों के योगदान का मूल्यांकन करता है। अतः विकल्प A सही है।

प्रश्न 24. भारत में औद्योगिक उत्पादन सूचकांक (IIP) के संबंध में निम्नलिखित कथनों पर विचार कीजिये:

  1. IIP एक समग्र संकेतक है जो औद्योगिक उत्पादों की एक टोकरी के उत्पादन की मात्रा में अल्पकालिक परिवर्तन को मापता है।
  2. IIP को सांख्यिकी और कार्यक्रम कार्यान्वयन मंत्रालय के राष्ट्रीय सांख्यिकी कार्यालय (NSO) द्वारा मासिक रूप से संकलित और प्रकाशित किया जाता है।

उपर्युक्त कथनों में से कौन-सा/से सही है/हैं?

(A) केवल 1
(B) केवल 2
(C) 1 और 2 दोनों
(D) उपरोक्त में से कोई नहीं 

उत्तर- C

व्याख्या: 

  • IIP एक संकेतक है जो एक निश्चित अवधि के दौरान औद्योगिक उत्पादों के उत्पादन की मात्रा में परिवर्तन को मापता है। अतः कथन 1 सही है।
  • यह एक समग्र संकेतक है जो निम्न के अंतर्गत वर्गीकृत उद्योग समूहों की विकास दर को मापता है:
    • व्यापक क्षेत्र अर्थात् खनन, विनिर्माण और बिजली।
    • उपयोग-आधारित क्षेत्र अर्थात् मूल वस्तुएँ, पूँजीगत वस्तुएँ और मध्यवर्ती वस्तुएँ।
  • इसे राष्ट्रीय सांख्यिकी कार्यालय (NSO), सांख्यिकी और कार्यक्रम कार्यान्वयन मंत्रालय द्वारा मासिक रूप से संकलित और प्रकाशित किया जाता है। अतः कथन 2 सही है।
    • IIP के लिये आधार वर्ष 2011-2012 है।

प्रश्न 25. क्रय प्रबंधक सूचकांक (PMI) के संबंध में निम्नलिखित कथनों पर विचार कीजिये:

  1. PMI एक समग्र संकेतक है जो विनिर्माण क्षेत्र के प्रदर्शन को मापता है।
  2. 50 से अधिक की रीडिंग विनिर्माण क्षेत्र में गिरावट को दर्शाती है, जबकि 50 से नीचे की रीडिंग विस्तार को दर्शाती है।

उपर्युक्त कथनों में से कौन-सा/से सही है/हैं?

(A) केवल 1
(B) केवल 2
(C) 1 और 2 दोनों
(D) उपरोक्त में से कोई नहीं 

उत्तर: A

व्याख्या:

  • क्रय प्रबंधक सूचकांक (PMI) एक समग्र संकेतक है जो विनिर्माण क्षेत्र के प्रदर्शन को मापता है। अतः कथन 1 सही है।
    • यह नए ऑर्डर, उत्पादन, रोज़गार, आपूर्तिकर्त्ता वितरण और सूची सहित क्षेत्र में वर्तमान आर्थिक परिस्थितियों का एक अवलोकन प्रदान करता है।
  • PMI पर 50 से ऊपर की रीडिंग विनिर्माण क्षेत्र में विस्तार को दर्शाती है, जबकि 50 से नीचे की रीडिंग गिरावट को दर्शाती है। अतः कथन 2 सही नहीं है।
  • PMI को आर्थिक गतिविधि का एक प्रमुख संकेतक माना जाता है, क्योंकि विनिर्माण क्षेत्र में परिवर्तन समग्र अर्थव्यवस्था में परिवर्तन से पहले होता है।

प्रश्न 26. उद्यमिता के संदर्भ में "फ्लिपिंग" क्या है?

(A) एक कंपनी के शेयरों को एक विदेशी संस्था को बेचने की प्रक्रिया
(B) एक भारतीय कंपनी के संपूर्ण स्वामित्व को एक विदेशी संस्था को स्थानांतरित करने की प्रक्रिया
(C) व्यवसाय का विस्तार करने के लिये एक विदेशी कंपनी का अधिग्रहण करने की प्रक्रिया
(D) स्टार्ट-अप के लिये सरकारी सहायता प्राप्त करने की प्रक्रिया

उत्तर: B

व्याख्या: 

  • उद्यमिता के संदर्भ में, "फ्लिपिंग"  बौद्धिक संपदा (IP) और भारतीय कंपनी के स्वामित्व वाले सभी डेटा के हस्तांतरण के साथ एक भारतीय कंपनी के संपूर्ण स्वामित्व को एक विदेशी संस्था को स्थानांतरित करने की प्रक्रिया को संदर्भित करता है।
  • यह प्रभावी रूप से एक भारतीय कंपनी को एक विदेशी इकाई की 100% सहायक कंपनी में बदल देता है, जिसके संस्थापक और निवेशक विदेशी इकाई के माध्यम से समान स्वामित्व बनाए रखते हैं तथा सभी शेयरों की अदला-बदली करते हैं।
  • यह प्रक्रिया प्राय: वाणिज्यिक, कराधान और संस्थापकों और निवेशकों की व्यक्तिगत प्राथमिकताओं द्वारा संचालित होती है। अतः विकल्प B सही है।

प्रश्न. 27 निम्नलिखित कथनों पर विचार कीजिये:

  1. विश्व निवेश रिपोर्ट विश्व आर्थिक मंच द्वारा जारी की जाती है।
  2. विश्व निवेश रिपोर्ट 2022 के अनुसार, वर्ष 2021 में शीर्ष 20 मेज़बान देशों में भारत FDI का तीसरा सबसे बड़ा प्राप्तकर्त्ता है।

उपर्युक्त कथनों में से कौन-सा/से सही है/हैं?

(A) केवल 1
(B) केवल 2
(C) 1 और 2 दोनों
(D) न तो 1 और न ही 2

उत्तर: D

  • UNCTAD की विश्व निवेश रिपोर्ट 2022 के अनुसार, वर्ष 2021 में शीर्ष 20 मेज़बान देशों में भारत FDI का सातवाँ सबसे बड़ा प्राप्तकर्त्ता है। अतः कथन 1 और 2 सही नहीं हैं।
  • FY22 में भारत ने सेवा क्षेत्र में 7.1 बिलियन अमेरिकी डॉलर FDI इक्विटी प्रवाह सहित 84.8 बिलियन अमेरिकी डॉलर का उच्चतम FDI प्राप्त किया।
  • निवेश की सुविधा के लिये सरकार द्वारा विभिन्न उपाय किये गए हैं, जैसे कि राष्ट्रीय एकल-खिड़की प्रणाली का शुभारंभ, निवेशकों, उद्यमियों और व्यवसायों द्वारा आवश्यक अनुमोदन और मंज़ूरी के लिये वन-स्टॉप समाधान।

प्रश्न. 28 विभिन्न उद्योगों में निवेश के उदारीकरण को सुनिश्चित करने के लिये सरकार द्वारा किये गए सुधारों के संदर्भ में निम्नलिखित कथनों पर विचार कीजिये:

  1. सरकार ने स्वचालित मार्ग के तहत जीवन बीमा निगम (LIC) और दूरसंचार सेवाओं में 100% विदेशी निवेश की अनुमति दी है।
  2. स्वचालित मार्ग के तहत बीमा कंपनियों में FDI की सीमा भी 49 से बढ़ाकर 74% कर दी गई।

उपर्युक्त कथनों में से कौन-सा/से सही है/हैं?

(A) केवल 1
(B) केवल 2
(C) 1 और 2 दोनों
(D) न तो 1 और न ही 2

उत्तर: B

  • विभिन्न उद्योगों में निवेश के उदारीकरण को सुनिश्चित करने के लिये सरकार ने स्वचालित मार्ग के माध्यम से सभी सेवाओं और बुनियादी ढाँचा प्रदाताओं सहित दूरसंचार सेवाओं में 100 प्रतिशत विदेशी भागीदारी की अनुमति दी है।
  • इसके अलावा सरकार ने स्वचालित मार्ग के तहत जीवन बीमा निगम (LIC) में 20% विदेशी निवेश की अनुमति दी है। अतः कथन 1 सही नहीं है।
  • स्वचालित मार्ग के तहत बीमा कंपनियों में FDI की सीमा भी 49 से बढ़ाकर 74 प्रतिशत कर दी गई। अतः कथन 2 सही है।

प्रश्न. 29 निम्नलिखित कथनों पर विचार कीजिये:

  1. भारतीय बीमा बाज़ार दुनिया का 10वाँ सबसे बड़ा बाज़ार है।
  2. भारतीय रिज़र्व बैंक भारत का बीमा नियामक है।

उपर्युक्त कथनों में से कौन-सा/से सही है/हैं?

(A) केवल 1
(B) केवल 2
(C) 1 और 2 दोनों
(D) न तो 1 और न ही 2

उत्तर: A

  • भारतीय बीमा क्षेत्र एक महत्त्वपूर्ण मोड़ पर है। भारत अगले दशक में वैश्विक बीमा उद्योग के विकास के मुख्य चालकों में से एक होगा। भारतीय बीमा बाज़ार दुनिया में 10वाँ सबसे बड़ा बाज़ार है एवं वर्ष 2032 तक जर्मनी, कनाडा, इटली और दक्षिण कोरिया से आगे 6वाँ सबसे बड़ा बाज़ार बनने की ओर अग्रसर है। अतः कथन 1 सही है।
  • बीमा नियामक, IRDAI ने सार्वभौमिक बीमा का मिशन शुरू किया है, जिससे बीमा क्षेत्र में उल्लेखनीय वृद्धि होने की उम्मीद है, जैसे कि जब भारत वर्ष 2047 में अपनी स्वतंत्रता के 100 वर्ष मनाएगा तो प्रत्येक भारतीय के पास उपयुक्त जीवन, स्वास्थ्य और संपत्ति बीमा कवर साथ ही, हर उद्यम उपयुक्त बीमा समाधान द्वारा समर्थित होना चाहिये। अतः कथन 2 सही नहीं है।

प्रश्न. 30 चिकित्सा पर्यटन सूचकांक के संबंध में निम्नलिखित कथनों पर कीजिये:

  1. चिकित्सा पर्यटन सूचकांक FY21 में भारत दुनिया के शीर्ष 46 देशों में से 10वें स्थान पर है।
  2. यह विश्व स्वास्थ्य संगठन द्वारा जारी किया जाता है।

उपर्युक्त कथनों में से कौन-सा/से सही है/हैं?

(A) केवल 1
(B) केवल 2
(C) 1 और 2 दोनों
(D) न तो 1 और न ही 2

उत्तर: A

  • मेडिकल टूरिज्म एसोसिएशन द्वारा जारी चिकित्सा पर्यटन सूचकांक FY21 में भारत दुनिया के शीर्ष 46 देशों में से 10वें स्थान पर है। अतः कथन 1 सही है और 2 सही नहीं है।
  • भारत ने जिस तरह से कोविड की स्थिति को संभाला है और भविष्य के लिये भी खुद को तैयार किया है उससे भारत के मेडिकल इंफ्रास्ट्रक्चर पर विश्वास बढ़ा है।

प्रश्न. 31 निम्नलिखित में से कौन-सी योजना वैश्विक चिकित्सा पर्यटन बाज़ार के बड़े हिस्से को कवर करने में सहायता करेगी?

  1. पर्यटकों के लिये आयुष वीज़ा
  2. सतत् पर्यटन व ज़िम्मेदार यात्री अभियान के लिये राष्ट्रीय रणनीति
  3. स्वदेश दर्शन 2.0 योजना
  4. हील इन इंडिया

निम्नलिखित कूटों का प्रयोग कर सही उत्तर चुनिये:

(A) केवल 1 और 2
(B) केवल 1, 2 और 4
(C) केवल 1, 3 और 4
(D) 1, 2, 3 और 4

उत्तर: D

  • भारत ने विशेषीकृत पर्यटन हेतु एक गंतव्य के रूप में अपने पर्यटन स्थलों/सुविधाओं में सुधार करने का प्रयास किया है।
  • चिकित्सा उपचार के लिये भारत आने की इच्छा रखने वाले पर्यटकों के लिये आयुष वीज़ा जैसी हालिया पहल, सतत् पर्यटन और ज़िम्मेदार यात्री अभियान के लिये राष्ट्रीय रणनीति का शुभारंभ, स्वदेश दर्शन 2.0 योजना की शुरुआत एवं हील इन इंडिया (Heal in India) वैश्विक चिकित्सा पर्यटन बाज़ार के एक बड़े हिस्से को कवर करने में सहायता कर सकता है। अतः विकल्प D सही उत्तर है।

प्रश्न. 32 आतिथ्य उद्योग के लिये मूल्यांकन, जागरूकता और प्रशिक्षण के लिये प्रणाली (साथी) के संदर्भ में निम्नलिखित कथनों पर विचार कीजिये:

  1. इसे भारत की गुणवत्ता परिषद के सहयोग से स्वास्थ्य और परिवार कल्याण मंत्रालय द्वारा लॉन्च किया गया था।
  2. इसका उद्देश्य लॉकडाउन के बाद आवास और अन्य सेवाएँ प्रदान करते हुए कोविड-19 वायरस के प्रसार को आगे रोकना है।

उपर्युक्त कथनों में से कौन-सा/से सही है/हैं?

(A) केवल 1
(B) केवल 2
(C) 1 और 2 दोनों
(D) न तो 1 और न ही 2

उत्तर: B

  • आतिथ्य उद्योग के लिये मूल्यांकन, जागरूकता और प्रशिक्षण के लिये प्रणाली (साथी) को पर्यटन मंत्रालय ने भारत की गुणवत्ता परिषद के साथ मिलकर आवास और अन्य सेवाएँ पोस्ट-लॉकडाउन में प्रदान करते हुए वायरस के किसी भी संचरण को प्रतिबंधित करने के लिये लॉन्च किया था। अतः कथन 1 सही नहीं है और कथन 2 सही है।
  • योजना का उद्देश्य सरकार के कोविड-19 नियमों पर उद्योग को संवेदनशील बनाना और कर्मचारियों एवं अतिथियों के बीच यह विश्वास पैदा करना है कि आतिथ्य इकाई ने कार्यस्थल पर सुरक्षा और स्वच्छता सुनिश्चित करने का संकल्प लिया है।

प्रश्न. 33 राष्ट्रीय आवास बैंक के संदर्भ में निम्नलिखित कथनों पर विचार कीजिये:

  1. NHB कंपनी अधिनियम, 1956 के तहत स्थापित एक वैधानिक संगठन है।
  2. यह एक सरकारी स्वामित्व वाली इकाई है।
  3. यह देश में आवास क्षेत्र के लिये शीर्ष स्तर की वित्तीय संस्था है।

उपर्युक्त कथनों में से कौन-सा/से सही है/हैं?

(A) केवल 1 और 2
(B) केवल 2 और 3
(C) केवल 3
(D) 1, 2 और 3

उत्तर: B

  • RBI की विशेष तरलता सुविधा के तहत, राष्ट्रीय आवास बैंक (NHB) ने महामारी की पहली और दूसरी लहर के दौरान क्रमशः 13,917 करोड़ रुपए और 8,112 करोड़ रुपए वितरित किये, ताकि क्षेत्र में हमेशा की तरह निर्बाध कारोबार सुनिश्चित किया जा सके। इसके साथ राष्ट्रीय आवास बैंक ने महामारी की शुरुआत के बाद से विभिन्न पुनर्वित्त योजनाओं के माध्यम से ₹ 88,400 करोड़ की तरलता सहायता प्रदान की है।
  • राष्ट्रीय आवास बैंक (NHB) एक वैधानिक संगठन है जिसे 9 जुलाई, 1988 को राष्ट्रीय आवास बैंक अधिनियम, 1987 के तहत स्थापित किया गया था।
  • यह देश में आवास क्षेत्र के लिये शीर्ष स्तर की वित्तीय संस्था है।
  • यह एक सरकारी स्वामित्व वाली इकाई है।
    • सरकार ने वर्ष 2019 में 1,450 करोड़ रुपए की पूरी हिस्सेदारी खरीदने के बाद RBI से NHB को अपने अंतर्गत ले लिया।
    • यह कदम अक्तूबर, 2001 की नरसिम्हम-द्वितीय समिति की रिपोर्ट की सिफारिश के बाद उठाया गया है।
    • इससे पहले RBI ने नाबार्ड में भी अपनी हिस्सेदारी बेची थी।
  • NHB का उद्देश्य हाउसिंग फाइनेंस संस्थानों के प्रचार को सुगम बनाना है और ऐसे संस्थानों को वित्तीय और अन्य सहायता प्रदान करना है।

प्रश्न. 34 नियोबैंक के संबंध में निम्नलिखित कथनों पर विचार कीजिये:

  1. वे कई शाखाओं वाले डिजिटल बैंक हैं।
  2. वे वित्तीय संस्थान हैं जो ग्राहकों को पारंपरिक बैंकों का सस्ता विकल्प देते हैं।
  3. इन बैंकों को भारतीय रिज़र्व बैंक (RBI) ने मंज़ूरी दे दी है।

उपर्युक्त कथनों में से कौन-सा/से सही है/हैं?

(A) केवल 1 और 2
(B) केवल 2
(C) केवल 2 और 3
(D) 1, 2 और 3

उत्तर: B

  • नियोबैंक एक तरह का डिजिटल बैंक है जिसकी कोई शाखा नहीं है। किसी विशिष्ट स्थान पर भौतिक रूप से उपस्थित होने के बजाय, नियो बैंकिंग पूरी तरह से ऑनलाइन है। अतः कथन 1 सही नहीं है।
  • नियोबैंक एक वित्तीय संस्थान हैं जो ग्राहकों को पारंपरिक बैंकों का सस्ता विकल्प देते हैं। अतः कथन 2 सही है।
  • वे परिचालन लागत को कम करते हुए ग्राहकों को व्यक्तिगत सेवाएँ प्रदान करने के लिये प्रौद्योगिकी और कृत्रिम बुद्धिमत्ता का लाभ उठाते हैं।
  • नियोबैंक ने 'चैलेंजर बैंक' के टैग के साथ वित्तीय प्रणाली में प्रवेश किया क्योंकि उन्होंने पारंपरिक बैंकों के जटिल बुनियादी ढाँचे और क्लाइंट ऑनबोर्डिंग प्रक्रिया को चुनौती दी थी।
  • भारत में इन फर्मों के पास स्वयं का बैंक लाइसेंस नहीं है लेकिन वे लाइसेंस प्राप्त सेवाओं की पेशकश के लिये बैंक भागीदारों पर निर्भर हैं।
    • इन बैंकों को अभी तक भारतीय रिज़र्व बैंक (RBI) द्वारा अनुमोदित नहीं किया गया है। अतः कथन 3 सही नहीं है।

प्रश्न. 35 सेंट्रल बैंक डिजिटल करेंसी (CBDC) के संबंध में निम्नलिखित कथनों पर विचार कीजिये:

  1. RBI ने केवल रिटेल सेगमेंट में CBDC को पायलट प्रोजेक्ट के रूप में लॉन्च किया है।
  2. यह फिएट करेंसी के समान है और फिएट करेंसी के साथ वन-टू-वन विनिमेय है।
  3. बहामा अपना राष्ट्रव्यापी CBDC लॉन्च करने वाली पहली अर्थव्यवस्था है।

उपर्युक्त कथनों में से कौन-सा/से सही है/हैं?

(A) केवल 1 और 2
(B) केवल 2
(C) केवल 2 और 3
(D) 1, 2 और 3

उत्तर: C

  • RBI ने हाल ही में होलसेल और रिटेल दोनों सेगमेंट में CBDC के पायलट प्रोजेक्ट लॉन्च किये हैं। अतः कथन 1 सही नहीं है।
  • डिजिटल रुपी-होलसेल- होलसेल के लिये पायलट प्रोजेक्ट 1 नवंबर, 2022 को लॉन्च किया गया था, जिसमें सरकारी प्रतिभूतियों में द्वितीयक बाज़ार लेनदेन के निपटान तक उपयोग का मामला सीमित था। डिजिटल रुपए के उपयोग से होलसेल अंतर-बैंक बाज़ार के और अधिक कुशल होने की उम्मीद है। रिटेल सेगमेंट में पायलट, जिसे डिजिटल रुपी-रिटेल के रूप में जाना जाता है, 1 दिसंबर, 2022 को भाग लेने वाले ग्राहकों और व्यापारियों के एक सीमित उपयोगकर्त्ता समूह के भीतर शुरू हुआ।
  • यह फिएट करेंसी के समान है और फिएट करेंसी के साथ वन-टू-वन विनिमेय है। अतः कथन 2 सही है।
  • फिएट करेंसी एक राष्ट्रीय मुद्रा है जो सोने या चांदी जैसी किसी वस्तु की कीमत से जुड़ी नहीं है।
  • बहामा अपना राष्ट्रव्यापी CBDC- सैंड डॉलर लॉन्च करने वाली पहली अर्थव्यवस्था है। अतः कथन 3 सही है।

प्रश्न. 36 अकाउंट एग्रीगेटर (AA) के संदर्भ में निम्नलिखित कथनों पर विचार कीजिये:

  1. यह एक गैर-बैंकिंग वित्तीय कंपनी है।
  2. यह ग्राहक से संबंधित वित्तीय जानकारी को पुनः प्राप्त करने या एकत्र करने की सेवा प्रदान करने के व्यवसाय में लगी हुई है।
  3. AA वित्तीय सूचना प्रदाताओं (FIP) और वित्तीय सूचना उपयोगकर्त्ताओं (FIU) के बीच डेटा के प्रवाह को सक्षम बनाते हैं।

उपर्युक्त कथनों में से कौन-सा/से सही है/हैं?

(A) केवल 1
(B) केवल 1 और 2
(C) केवल 2 और 3
(D) 1, 2 और 3

उत्तर: D

  • अकाउंट एग्रीगेटर (AA) एक गैर-बैंकिंग वित्तीय कंपनी (NBFC) है जो ग्राहक से संबंधित वित्तीय जानकारी प्राप्त करने या एकत्र करने की सेवा प्रदान करने के व्यवसाय में लगी हुई है। अतः कथन 1 और 2 सही हैं।
  • ग्राहक की स्पष्ट सहमति के बिना AA द्वारा ग्राहक की कोई भी वित्तीय जानकारी पुनर्प्राप्त, साझा या स्थानांतरित नहीं की जाती है।
  • AA किसी व्यक्ति के निर्देश और सहमति के आधार पर डेटा को एक वित्तीय संस्थान से दूसरे में स्थानांतरित करता है। उपभोक्ताओं के लिये AA के साथ पंजीकरण पूरी तरह से स्वैच्छिक है।
  • संस्थाएँ वित्तीय सूचना प्रदाता (FIP) के रूप में AA ढाँचे पर खुद को नामांकित कर सकती हैं। बैंकिंग कंपनी, गैर-बैंकिंग वित्तीय कंपनी, परिसंपत्ति प्रबंधन कंपनी, डिपॉजिटरी, डिपॉजिटरी पार्टिसिपेंट, बीमा कंपनी, बीमा रिपॉजिटरी, पेंशन फंड आदि और वित्तीय सूचना उपयोगकर्त्ता (FIU) के रूप में जो किसी वित्तीय क्षेत्र के नियामक के साथ पंजीकृत और विनियमित संस्था है।
  • AA FIP और FIU के बीच डेटा के प्रवाह को सक्षम करते हैं। अतः कथन 3 सही है।

प्रश्न. 37 नेशनल ई-गवर्नेंस सर्विसेज लिमिटेड (NeSL) के संदर्भ में निम्नलिखित कथनों पर विचार कीजिये:

  1. NeSL भारत की पहली इनफॉर्मेशन यूटिलिटी है।
  2. NeSL दिवाला और शोधन अक्षमता संहिता, 2016 (IBC) के तत्त्वावधान में भारतीय दिवाला और शोधन अक्षमता बोर्ड (IBBI) के साथ पंजीकृत है।
  3. NeSL ने डिजिटल डॉक्यूमेंट एक्ज़ीक्यूशन (DDE) प्लेटफॉर्म लॉन्च किया, जिसका उद्देश्य दस्तावेज़/ अनुबंध निष्पादन यात्रा के सभी चरणों को डिजिटलीकरण करना है।

उपर्युक्त कथनों में से कौन-सा/से सही है/हैं?

(A) केवल 1 और 2
(B) केवल 2 और 3
(C) केवल 3
(D) 1, 2 और 3

उत्तर: D

  • नेशनल ई-गवर्नेंस सर्विसेज़ (NeSl) की प्राथमिक भूमिका किसी भी ऋण/दावे से संबंधित जानकारी रखने वाले कानूनी साक्ष्य के भंडार के रूप में कार्य करना है, जैसा कि वित्तीय या परिचालन लेनदार द्वारा प्रस्तुत किया जाता है और पार्टियों द्वारा ऋण के लिये सत्यापित और प्रमाणित किया जाता है।
  • NeSL भारत की पहली इनफॉर्मेशन यूटिलिटी है एवं दिवाला और शोधन अक्षमता संहिता, 2016 (IBC) के तत्त्वावधान में भारतीय दिवाला और शोधन अक्षमता बोर्ड (IBBI) के साथ पंजीकृत है। अतः कथन 1 और 2 सही हैं।
  • NeSL ने डिजिटल डॉक्यूमेंट एक्ज़ीक्यूशन (DDE) प्लेटफॉर्म लॉन्च किया, जिसका उद्देश्य दस्तावेज़/ अनुबंध निष्पादन यात्रा के सभी चरणों को डिजिटलीकरण करना है तथा इसने अपने डिजिटल दस्तावेज़ निष्पादन (DDE) प्लेटफॉर्म के माध्यम से एक मिलियन लेनदेन की प्रक्रिया की है। अतः कथन 3 सही है।

प्रश्न 38. निम्नलिखित कथनों पर विचार कीजिये:

  1. 2017-18 के बाद से सकल घरेलू उत्पाद के प्रतिशत में वृद्धि के कारण सरकार द्वारा सामाजिक सेवाओं पर व्यय की प्रवृत्ति देखी गई है।
  2. 2017-18 के बाद से सरकार के कुल राजस्व व्यय के प्रतिशत में निरंतर वृद्धि के कारण सरकार द्वारा सामाजिक सेवाओं पर व्यय की प्रवृत्ति देखी गई है।

नीचे दिये गए कूट से सही उत्तर का चयन कीजिये:

(A) केवल 1
(B) केवल 2 
(C) 1 और 2 दोनों 
(D) न तो 1 न ही 2

उत्तर: A

व्याख्या:

  • यदि हम निम्नलिखित ग्राफ को देखें, तो हम पाते हैं कि सकल घरेलू उत्पाद के प्रतिशत के रूप में सामाजिक सेवाओं पर सरकार के व्यय में 2017-18 के बाद से वृद्धि की प्रवृत्ति दिखाई दे रही है। लेकिन वर्ष 2020-2021 में सरकार के कुल राजस्व व्यय के प्रतिशत के रूप में सामाजिक सेवाओं पर सरकार के व्यय में कमी आई है। अतः कथन 1 सही है परंतु कथन 2 सही नहीं है।

प्रश्न 39. मानव विकास सूचकांक (HDI) पर संयुक्त राष्ट्र विकास कार्यक्रम (UNDP) की रिपोर्ट के संदर्भ में निम्नलिखित कथनों पर विचार कीजिये:

  1. 32 वर्ष में पहली बार विश्वभर में 2020 या 2021 में मानव विकास में गिरावट दर्ज़ की गई है।
  2. 2021 में भारत का मानव विकास सूचकांक मान देश को मध्यम मानव विकास श्रेणी में रखता है।
  3. भारत का मानव विकास सूचकांक मान दक्षिण एशिया के औसत मानव विकास से कम है।

उपर्युक्त कथनों में से कौन-सा/से सही है/हैं?

(A) केवल 1 और 2 
(B) केवल 1 और 3 
(C) केवल 2 और 3
(D) 1, 2 और 3

उत्तर: A

व्याख्या:

  • 'मानव विकास' उर्ध्वगामी सामाजिक गतिशीलता के लिये प्रमुख प्रवर्तक है। वर्ष 2020-2021 में कोविड -19 महामारी के चरम के बाद की चुनौतियाँ और वर्ष 2022 में रूस-यूक्रेन संघर्ष ने भारत तथा विश्व के विकास को प्रभावित किया है। 
  • इस घटनाओं के कारण, मानव विकास में वैश्विक गिरावट दर्ज़ की गई थी। संयुक्त राष्ट्र विकास कार्यक्रम (UNDP) की रिपोर्ट के अनुसार, 90 प्रतिशत देशों ने वर्ष 2020 या वर्ष 2021 में अपने मानव विकास सूचकांक (HDI) मान में कमी दर्ज़ की है, यह दर्शाता है कि विश्वभर में मानव विकास 32 वर्षों में पहली बार अवरुद्ध हुआ है। अतः कथन 1 सही है।
  • वर्ष 2021-2022 के मानव विकास सूचकांक रिपोर्ट में भारत 191 देशों और क्षेत्रों में से 132वें स्थान पर है। वर्ष 2021 में भारत का मानव विकास सूचकांक मान 0.633 था, जो 2019 में इसके 0.645 के मान से कम है, जो देश को मध्यम मानव विकास श्रेणी में रखता है। अतः कथन 2 सही है।
  • हालाँकि, भारत का मानव विकास सूचकांक (HDI) मान दक्षिण एशिया के औसत मानव विकास से अधिक है। अतः कथन 3 सही नहीं है।
    • सार्वभौमिक स्वास्थ्य और शिक्षा सुनिश्चित करने सहित सामाजिक अवसंरचनात्मक ढाँचे में निवेश को प्राथमिकता देने के कारण वर्ष 1990 के बाद से यह निरंतर विश्व औसत की ओर बढ़ रहा है। 

प्रश्न 40. 'आकांक्षी ज़िलों का रूपांतरण' कार्यक्रम के संबंध में निम्नलिखित कथनों पर विचार कीजिये:

  1. इसका लक्ष्य अपने नागरिकों के जीवन स्तर को ऊपर उठाना और बढ़ती अर्थव्यवस्था में सभी के समावेशी विकास को सुनिश्चित करना है।
  2. समग्र संकेतकों के आधार पर भारत सरकार द्वारा 28 राज्यों/केंद्र शासित प्रदेशों में 117 आकांक्षी ज़िलों (AD) की पहचान की गई है।
  3. ज़िलों का क्रम-निर्धारण हर महीने 49 प्रमुख प्रदर्शन संकेतकों (KPI) में हुई क्रमिक प्रगति पर आधारित है।

उपर्युक्त कथनों में से कौन-से सही हैं?

(A) केवल 1 और 2 
(B) केवल 1 और 3 
(C) केवल 2 और 3 
(D) 1, 2 और 3

उत्तर: B

व्याख्या:

  • भारत सरकार ने जनवरी 2018 में 'आकांक्षी ज़िलों का रूपांतरण' (आकांक्षी ज़िला कार्यक्रम) पहल की शुरुआत वर्ष 2022 तक नए भारत के लक्ष्य के साथ किया, जिसमें अपने नागरिकों के जीवन स्तर को बढ़ाने और सभी का समावेशी विकास सुनिश्चित करने पर ध्यान केंद्रित किया गया है। अतः कथन 1 सही है।
  • स्वास्थ्य और पोषण, शिक्षा, कृषि और जल संसाधन, वित्तीय समावेशन और कौशल विकास तथा अवसंरचनात्मक ढाँचे से लेकर मानव विकास सूचकांक (HDI) को प्रभावित करने वाले समग्र संकेतकों के आधार पर नीति आयोग द्वारा 28 राज्यों/केंद्र शासित प्रदेशों में 117 आकांक्षी ज़िलों (AD) की पहचान की गई है। अतः कथन 2 सही नहीं है।
  • मुख्य उत्प्रेरकों के रूप में राज्यों के साथ, यह कार्यक्रम प्रत्येक ज़िले की क्षमता पर ध्यान केंद्रित करता है, तत्काल सुधार के लिये आसान और सुगम लक्ष्यों की पहचान करता है तथा हर महीने ज़िलों का क्रम-निर्धारण करके प्रगति को मापता है। क्रम-निर्धारण ऊपर वर्णित पाँच व्यापक सामाजिक-आर्थिक विषयों के तहत 49 प्रमुख प्रदर्शन संकेतक (KPI) में हुई वार्धिक प्रगति पर आधारित है। अतः कथन 3 सही है।

प्रश्न 41. ई-श्रम पोर्टल के संदर्भ में, निम्नलिखित कथनों पर विचार कीजिये:

  1. इलेक्ट्रॉनिक्स और सूचना प्रौद्योगिकी मंत्रालय ने असंगठित श्रमिकों का एक राष्ट्रीय डेटाबेस बनाने के लिये पोर्टल विकसित किया है, जो आधार से सत्यापित है।
  2. इस राष्ट्रीय डेटाबेस में प्रवासी श्रमिक, निर्माण श्रमिक, गिग और प्लेटफॉर्म श्रमिक आदि शामिल हैं।
  3. महिला पंजीकरण पुरुष पंजीकरण से अधिक है।

उपर्युक्त कथनों में से कौन-से सही हैं?

(A) केवल 1 और 2 
(B) केवल 1 और 3 
(C) केवल 2 और 3 
(D) 1, 2 और 3

उत्तर: C

व्याख्या:

  • तद्नुसार, श्रम एवं रोजगार मंत्रालय (MoLE) ने असंगठित श्रमिकों का एक राष्ट्रीय डेटाबेस बनाने के लिये ई-श्रम पोर्टल विकसित किया है, जो आधार से सत्यापित है। अतः कथन 1 सही नहीं है।
  • यह श्रमिकों के नाम, व्यवसाय, पता, व्यवसाय प्रकार, शैक्षिक योग्यता और कौशल प्रकार आदि जैसे विवरणों को उनकी रोज़गार क्षमता के इष्टतम साधन के लिये प्रग्रहित करता है और उनके सामाजिक सुरक्षा योजनाओं के लाभों का विस्तार करता है। 
  • यह प्रवासी श्रमिकों, निर्माण श्रमिकों, गिग और प्लेटफॉर्म श्रमिकों आदि सहित असंगठित श्रमिकों का पहला राष्ट्रीय डेटाबेस है। अतः कथन 2 सही है।
  • वर्तमान में सेवाओं की सहज सुविधा के लिये ई-श्रम पोर्टल को NCS पोर्टल और ASEEM पोर्टल से जोड़ा गया है।
  • 31 दिसंबर 2022 तक, कुल 28.5 करोड़ से अधिक असंगठित श्रमिकों को ई-श्रम पोर्टल पर पंजीकृत किया गया है। महिला पंजीकरण कुल पंजीकरण का 52.8 प्रतिशत था और कुल पंजीकरण का 61.7 प्रतिशत 18-40 वर्ष की आयु वर्ग का था। अतः कथन 3 सही है।
  • राज्य-वार, उत्तर प्रदेश (29.1 प्रतिशत), बिहार (10.0 प्रतिशत) और पश्चिम बंगाल (9.0 प्रतिशत) से कुल पंजीकरण का लगभग आधा हिस्सा हैं। 
  • कृषि क्षेत्र के श्रमिकों ने कुल पंजीकरण में 52.4 प्रतिशत का योगदान दिया, इसके बाद घरेलू और घरेलू श्रमिकों (9.8 प्रतिशत) और निर्माण श्रमिकों (9.1 प्रतिशत) का स्थान रहा।

प्रश्न 42. प्रधानमंत्री फसल बीमा योजना (PMFBY) के संबंध में निम्नलिखित कथनों पर विचार कीजिये: 

  1. PMFBY को वर्ष 2016 के खरीफ मौसम में फसल खराब होने की स्थिति में किसानों को व्यापक बीमा कवरेज प्रदान करने के लिये शुरू किया गया था, जिससे किसानों की आय स्थिर हुई। 
  2. PMFBY वर्तमान में किसान नामांकन के मामले में विश्व की सबसे बड़ी फसल बीमा योजना है।

उपर्युक्त कथनों में से कौन-सा/से सही है/हैं? 

(A) केवल 1 
(B) केवल 2 
(C) 1 और 2 दोनों   
(D) न तो 1 न ही 2 

उत्तर: C 

व्याख्या: 

  • PMFBY को वर्ष 2016 के खरीफ मौसम के दौरान किसानों को फसल खराब होने की स्थिति में पूर्ण बीमा कवरेज देने और उनकी आय को स्थिर करने के लिये प्रस्तावित किया गया था। योजना के कार्यान्वयन हेतु सामान्य बीमा कंपनियाँ नियुक्त की गई। वर्तमान राजनीतिक परिदृश्य और राष्ट्र की फसल बीमा में नीतिगत भागीदारी की आवश्यकता को देखते हुए इसे समयबद्ध आधार पर उन्नत/संशोधित किया जाता है। इस योजना में सभी खाद्य और तिलहन फसलों के साथ-साथ वार्षिक वाणिज्यिक और बागवानी फसलें भी शामिल हैं, जिनके लिये विगत् उपज की जानकारी उपलब्ध है और जिसके लिये सामान्य फसल अनुमान सर्वेक्षण (GCES) के हिस्से के रूप में आवश्यक संख्या में फसल कटाई प्रयोग (CCE) आयोजित किये जा रहे हैं। सभी किसान खरीफ 2020 से शुरू होने वाली अद्यतन योजना में भाग लेने के लिये स्वतंत्र हैं और राज्यों ने लगातार तीन वर्षों के लिये बीमा कंपनियों का चयन किया है। अतः कथन 1 सही है। 
  • किसानों की भागीदारी के मामले में, PMFBY वर्तमान में विश्वभर में नामांकन करने वाला सबसे बड़ा फसल बीमा कार्यक्रम है, जिसमें हर वर्ष औसतन 5.5 करोड़ किसान आवेदन करते हैं और प्राप्त प्रीमियम के मामले में तीसरे स्थान पर है। 31 अक्टूबर 2022 तक प्राप्त जानकारी के अनुसार किसानों ने इसके कार्यान्वयन के विगत् छह वर्षों के दौरान 25,186 अरब रुपये प्रीमियम का भुगतान किया है और कुल 1.26 लाख अरब रुपये का दावा किया है। अतः कथन 2 सही है।

प्रश्न 43. मिलेट्स/ मोटे अनाज के संदर्भ में निम्नलिखित कथनों पर विचार कीजिये: 

  1. संयुक्त राष्ट्र महासभा ने मार्च 2021 के दौरान अपने 75वें सत्र में वर्ष 2023 को अंतर्राष्ट्रीय मिलेट्स वर्ष (IYM) घोषित किया।
  2. भारत में मिलेट्स या मोटे अनाज मुख्य रूप से रबी फसल है। 
  3. भारत मिलेट्स या मोटे अनाज का 50.9 मिलियन टन (चौथे अग्रिम अनुमान के अनुसार) से अधिक उत्पादन करता है जो एशिया के कुल उत्पादन का 80 प्रतिशत और वैश्विक उत्पादन का 20 प्रतिशत है। 

उपर्युक्त कथनों में से कौन-से सही हैं? 

(A) केवल 1 और 2 
(B) केवल 2 और 3 
(C) केवल 3 और 1 
(D) 1, 2 और 3 

उत्तर: C 

व्याख्या: 

  • संयुक्त राष्ट्र महासभा ने मार्च 2021 में अपने 75वें सत्र के दौरान वर्ष 2023 को अंतर्राष्ट्रीय मिलेट्स वर्ष (IYM) घोषित किया। मिलेट्स या मोटे अनाज उच्च पोषण मूल्य वाला एक खाद्य पदार्थ है, जो जलवायु समुत्थानशील है और कई संयुक्त राष्ट्र के सतत् विकास लक्ष्यों (SDG) के साथ संरेखित है। ये इसलिये भी महत्त्वपूर्ण हैं क्योंकि इनमें आजीविका उत्पन्न करने, किसानों की आय बढ़ाने तथा वैश्विक खाद्य और पोषण सुरक्षा सुनिश्चित करने की अपार क्षमता है। अतः कथन 1 सही है। 
  • भारत में, मिलेट्स या मोटे अनाज मुख्य रूप से खरीफ फसल है, जो ज़्यादातर वर्षा आधारित परिस्थितियों में उगाई जाती है, जिसमें अन्य प्रमुख फसलों की तुलना में कम जल और कृषि आदानों की आवश्यकता होती है। अतः कथन 2 सही नहीं है। 
  • भारत 50.9 मिलियन टन से अधिक मिलेट्स या मोटे अनाज (चौथे अग्रिम अनुमान के अनुसार) का उत्पादन करता है, जो एशिया के कुल उत्पादन का 80% और कुल वैश्विक उत्पादन का 20% है। वैश्विक औसत उपज 1229 किग्रा/हेक्टेयर है, लेकिन भारत की औसत उपज 1239 किग्रा/हेक्टेयर है। अतः कथन 3 सही है। 

प्रश्न 44. जन शिक्षण संस्थान योजना (JSS) के संबंध में निम्नलिखित कथनों पर विचार कीजिये:

  1. शिक्षा मंत्रालय JSS के लिये नोडल मंत्रालय है।
  2. JSS का मुख्य उद्देश्य गैर-साक्षर, नव-साक्षर, साथ ही स्कूल छोड़ने वालों को उनकी ज़रूरतों के अनुसार कौशल की पहचान करके व्यावसायिक कौशल प्रदान करना है।

उपर्युक्त कथनों में से कौन-सा/से सही है/हैं?

(A) केवल 1 
(B) केवल 2 
(C) 1 और 2 दोनों 
(D) उपरोक्त में से कोई नहीं 

उत्तर: b

व्याख्या: 

  • योजना को वर्ष 2018 में शिक्षा मंत्रालय से कौशल विकास और उद्यमशीलता मंत्रालय में स्थानांतरित कर दिया गया था। अतः कथन 1 सही नहीं है।
  • JSS का उद्देश्य गैर-साक्षर, नव-साक्षर, साथ ही स्कूल छोड़ने वालों को उनकी ज़रूरतों के अनुसार कौशल की पहचान करके व्यावसायिक कौशल प्रदान करना और साक्षर व्यक्तियों को व्यावसायिक कौशल प्रदान करना है, जिन्होंने शिक्षा के अवसरों को खो दिया है और उनकी आजीविका स्तर में वृद्धि करने हेतु कौशल विकास में सुधार की आवश्यकता है। अतः कथन 2 सही है।

प्रश्न 45. पीएम-श्री (PM SHRI) के संदर्भ में निम्नलिखित कथनों पर विचार कीजिये: 

  1. पीएम स्कूल फॉर राइजिंग इंडिया' (PM SHRI) मौजूदा सरकारी स्कूलों को मॉडल स्कूलों में बदलने की एक योजना है। 
  2. PM-SHRI का मुख्य उद्देश्य आर्थिक रूप से कमज़ोर वर्ग के छात्रों को मुफ्त शिक्षा और छात्रवृत्ति प्रदान करना है।

उपर्युक्त कथनों में से कौन-सा/से सही है/हैं? 

(A) केवल 1 
(B) केवल 2 
(C) 1 और 2 दोनों  
(D) उपरोक्त में से कोई नहीं  

उत्तर: A

व्याख्या: 

  • केंद्रीय मंत्रिमंडल ने मौजूदा सरकारी स्कूलों को मॉडल स्कूलों में बदलने के लिये 'पीएम स्कूल फॉर राइजिंग इंडिया' (PM SHRI) योजना को मंजूरी दी। यह योजना एक समान, समावेशी वातावरण में उच्च गुणवत्ता वाली शिक्षा प्रदान करेगी जो विविध पृष्ठभूमि, बहुभाषी आवश्यकताओं और बच्चों की विभिन्न शैक्षणिक क्षमताओं का ध्यान रखेगी। अतः कथन 1 सही है।
  • PM-SHRI का मुख्य उद्देश्य एक समान, समावेशी वातावरण में उच्च गुणवत्ता वाली शिक्षा प्रदान करना है जो विविध पृष्ठभूमि, बहुभाषी आवश्यकताओं और बच्चों की विभिन्न शैक्षणिक क्षमताओं का ध्यान रखता है। अतः कथन 2 सही नहीं है।

प्रश्न 46. निम्नलिखित में कौन-सा मंत्रालय भारत में उद्योगों का वार्षिक सर्वेक्षण (ASI) आयोजित करता है? 

(A) वाणिज्य और उद्योग मंत्रालय
(B) सांख्यिकी और कार्यक्रम कार्यान्वयन मंत्रालय
(C) सूक्ष्म, लघु और मध्यम उद्यम मंत्रालय
(D) वित्त मंत्रालय 

उत्तर: B

व्याख्या:

  • सांख्यिकी और कार्यक्रम कार्यान्वयन मंत्रालय के केंद्रीय सांख्यिकी संगठन (CSO) का औद्योगिक सांख्यिकी प्रभाग सर्वेक्षण पद्धति की रूपरेखा तैयार करने, डेटा के प्रसंस्करण और रिपोर्ट तैयार करने के लिये ज़िम्मेदार है। अतः विकल्प B सही है।

प्रश्न 47. राष्ट्रीय कॅरियर सेवा परियोजना के संदर्भ में, निम्नलिखित कथनों पर विचार कीजिये:

  1. यह सरकार द्वारा संचालित रोज़गार अभिकरण है।
  2. 2 करोड़ से अधिक नौकरी चाहने वाले लोग राष्ट्रीय कॅरियर सेवा परियोजना के साथ पंजीकृत हैं।
  3. डिजी-सक्षम कार्यक्रम कॅरियर कौशल पर मुफ्त, स्व-केंद्रित ऑनलाइन/ऑफलाइन प्रशिक्षण प्रदान करता है।

उपर्युक्त कथनों में से कौन-से सही हैं?

(A) केवल 1 और 2 
(B) केवल 2 और 3 
(C) केवल 3 और 1 
(D) 1, 2 और 3 

उत्तर: b

व्याख्या:

  • राष्ट्रीय कॅरियर सेवा (NCS) भारत सरकार द्वारा जुलाई 2015 में शुरू की गई एक परियोजना है, जो रोज़गार और कॅरियर से संबंधित सेवाओं की एक शृंखला प्रदान करती है। इसका उद्देश्य प्रशिक्षण और कॅरियर मार्गदर्शन चाहने वाले उम्मीदवारों तथा प्रशिक्षण और कॅरियर परामर्श प्रदान करने वाली एजेंसियों के बीच की खाई को पाटना है। अतः कथन 1 सही नहीं है।
  • जनवरी 2023 तक, 2.8 करोड़ नौकरी चाहने वालों और 6.8 लाख नियोक्ताओं ने NCS पोर्टल पर पंजीकरण कराया है। अतः कथन 2 सही है।
  • डिजीसक्षम कार्यक्रम राष्ट्रीय कॅरियर सेवा (NCS) और निजी क्षेत्र के बीच एक साझेदारी है, जो नौकरी चाहने वालों को कॅरियर कौशल पर मुफ्त, स्व-केंद्रित ऑनलाइन/ऑफ़लाइन प्रशिक्षण प्रदान करता है। कार्यक्रम का उद्देश्य नौकरी चाहने वालों को उनकी रोज़गार क्षमता में सुधार के लिए सॉफ्ट और डिजिटल कौशल की एक श्रृंखला से युक्त करना है। अतः कथन 3 सही है।

प्रश्न 48. कौशल भारत मिशन के संदर्भ में, निम्नलिखित कथनों पर विचार कीजिये:

  1. कौशल भारत मिशन का मुख्य लक्ष्य स्किलिंग, री-स्किलिंग और अप-स्किलिंग है।
  2. राष्ट्रीय कौशल विकास निगम का उद्देश्य भारत को विश्व की कौशल राजधानी बनाना है।
  3. स्किल एक्विजिशन एंड नॉलेज अवेयरनेस फॉर लाइवलीहुड प्रमोशन (SANKALP) प्रोग्राम एशियाई विकास  बैंक द्वारा वित्तपोषित है।

उपर्युक्त कथनों में से कौन-सा/से सही है/हैं?

(A) केवल 1 और 2 
(B) केवल 2 
(C) केवल 3 
(D) 1, 2 और 3 

उत्तर: A

व्याख्या:

  • कौशल भारत मिशन स्किलिंग, री-स्किलिंग और अप-स्किलिंग हेतु अल्पकालिक एवं दीर्घकालिक प्रशिक्षण कार्यक्रम प्रदान करने पर केंद्रित है। अतः कथन 1 सही है।
  • भारत को विश्व की कौशल राजधानी बनाने और कुशल जनशक्ति की गतिशीलता में सुधार लाने के उद्देश्य से राष्ट्रीय कौशल विकास निगम इंटरनेशनल की स्थापना की गई है। अतः कथन 2 सही है।
  • स्किल एक्विजिशन एंड नॉलेज अवेयरनेस फॉर लाइवलीहुड प्रमोशन (SANKALP) वर्ष 2018 में शुरू किया गया एक विश्व बैंक ऋण-सहायता कार्यक्रम है, जिसका उद्देश्य कौशल पहलों को विकेंद्रीकृत करना तथा युवाओं को उनके स्थानीय मांग और आकांक्षाओं के साथ कौशल विकास कार्यक्रमों को संरेखित करना है। अतः कथन 3 सही नहीं है।

प्रश्न 49. निम्नलिखित कथनों पर विचार कीजिये:

  1. सरकार द्वारा शुरू किये गए विद्यांजलि कार्यक्रम का उद्देश्य वंचित छात्रों को छात्रवृत्ति प्रदान करना है।
  2. विद्यांजलि पोर्टल समुदाय और स्वयंसेवकों/संगठनों के लिये सरकारी और सरकारी सहायता प्राप्त स्कूलों के साथ संवाद करने और उनसे जुड़ने का एक पोर्टल है।

उपर्युक्त कथनों में से कौन-सा/से सही है/हैं?

(A) केवल 1 
(B) केवल 2 
(C) 1 और 2 दोनों 
(D) उपरोक्त में से कोई नहीं 

उत्तर: B

व्याख्या:

  • सरकार द्वारा शुरू किये गए विद्यांजलि कार्यक्रम का उद्देश्य समुदाय, कॉपोरेट सामाजिक उत्तरदायित्त्व (CSR) और निजी क्षेत्र की भागीदारी के माध्यम से स्कूलों को सुदृढ़  करना एवं स्कूली शिक्षा की गुणवत्ता में सुधार करना है। अतः कथन 1 सही नहीं है।
  • विद्यांजलि पोर्टल समुदाय और स्वयंसेवकों/संगठनों को उनकी पसंद के सरकारी और सरकारी सहायता प्राप्त स्कूलों के साथ संवाद करने और सीधे जुड़ने में सक्षम बनाता है। अतः कथन 2 सही है।

प्रश्न 50. राष्ट्रीय फसल बीमा पोर्टल के संदर्भ में निम्नलिखित कथनों पर विचार कीजिये:

  1. यह सभी शिकायतों के आद्योपांत प्रबंधन हेतु डिज़ाइन किया गया है
  2. इसमें बीमा कंपनियों के लिये API आधारित कनेक्टिविटी शामिल है।
  3. इसे वर्ष 2022 में लॉन्च किया गया था।
  4. इसका उद्देश्य फसलों की उपज में वृद्धि करना।

उपर्युक्त कथनों में से कौन-सा/से सही है/हैं?

(A) केवल 1 और 2
(B) केवल 2 और 4
(C) केवल 1,2 और 3
(D) 1, 2, 3 और 4

उत्तर: C

व्याख्या:

  • राष्ट्रीय फसल बीमा पोर्टल:
    • यह सभी शिकायतों के आद्योपांत प्रबंधन हेतु डिज़ाइन किया गया है, अतः कथन 1 सही है।
    • पोर्टल में बीमा कंपनियों के लिये API आधारित कनेक्टिविटी शामिल है। अतः कथन 2 सही है।
    • इसे 21 जुलाई, 2022 को छत्तीसगढ़ में बीटा संस्करण में लॉन्च किया गया था, अतः कथन 3 सही है।
    • राष्ट्रीय फसल बीमा पोर्टल का उद्देश्य फसल बीमा से संबंधित शिकायतों का प्रबंधन करना है न कि फसलों की उपज में वृद्धि करना। अतः कथन 4 सही नहीं है।

प्रश्न 51. प्रधानमंत्री फसल बीमा योजना (PMFBY) के संदर्भ में निम्नलिखित कथनों पर विचार कीजिये:

  1. इसे वर्ष 2016 के खरीफ मौसम में लॉन्च किया गया था।
  2. इसमें सभी खाद्य एवं तिलहन फसलें तथा वार्षिक वाणिज्यिक/बागवानी फसलें शामिल हैं।
  3. यह योजना 'क्षेत्र दृष्टिकोण' के आधार पर लागू की गई है।
  4. वर्ष 2017, 2018 और 2019 के दौरान एकत्र किये गए सकल प्रीमियम के मुकाबले कई राज्यों में दावों का भुगतान अनुपात औसतन 100% से अधिक रहा।

उपर्युक्त कथनों में से कौन-सा/से सही है/हैं?

(A) केवल 1, 2 और 3
(B) केवल 2, 3 और 4
(C) केवल 1 और 4
(D) उपरोक्त सभी

उत्तर: D

व्याख्या: 

  • प्रधानमंत्री फसल बीमा योजना (PMFBY):
    • प्रधानमंत्री फसल बीमा योजना (PMFBY) 13 जनवरी, 2016 को उस वर्ष के खरीफ मौसम हेतु शुरू की गई थी। अतः कथन 1 सही है।
    • PMFBY में सभी खाद्य एवं तिलहन फसलें तथा वार्षिक वाणिज्यिक/बागवानी फसलें शामिल हैं। अतः कथन 2 सही है।
    • यह योजना 'क्षेत्र दृष्टिकोण' आधार पर कार्यान्वित की जाती है, जिसका अर्थ है कि यह सभी किसानों के लिये एक संपूर्ण क्षेत्र या फसल को कवर करती है। यह दृष्टिकोण सुनिश्चित करता है कि किसी क्षेत्र विशेष के सभी किसान योजना के अंतर्गत आते हैं और उनके बीच कोई भेदभाव नहीं होता है। अतः कथन 3 सही है।
    • वर्ष 2017, 2018 और 2019 के कठिन सत्रों के दौरान एकत्र किये गए सकल प्रीमियम के मुकाबले कई राज्यों में दावों का भुगतान अनुपात औसतन 100% से अधिक रहा। इसका अर्थ है कि दावों के रूप में भुगतान की गई राशि इन मौसमों के दौरान प्रीमियम के रूप में एकत्र की गई राशि से अधिक थी।
      • इसका कारण यह है कि इन वर्षों के दौरान बाढ़, सूखा और चक्रवात जैसी प्राकृतिक आपदाओं के कारण कई फसल विफलताएँ हुईं, जिसके कारण अधिक संख्या में दावे दायर किये गए। सरकार को प्रीमियम के रूप में एकत्र किये गए दावों की तुलना में अधिक भुगतान करना पड़ा, जिसके परिणामस्वरूप दावा भुगतान अनुपात 100% से अधिक हो गया। अतः कथन 4 सही है।

प्रश्न 52. भारत का कौन-सा राज्य पूर्ण रूप से जैविक बनने वाला पहला राज्य था?

(A) उत्तराखंड
(B) सिक्किम
(C) त्रिपुरा
(D) मध्य प्रदेश

उत्तर: B 

व्याख्या: 

  • आर्थिक सर्वेक्षण में कहा गया है कि सिक्किम ने स्वेच्छा से जैविक खेती को अपनाया और जैविक खेती के तहत 58,168 हेक्टेयर की कुल कृषि योग्य भूमि प्राप्त करने की प्रक्रिया वर्ष 2010 में ज़मीनी स्तर पर शुरू हुई। यह पूरी तरह से जैविक बनने वाला दुनिया का पहला राज्य बन गया। अतः विकल्प B सही है।

प्रश्न 53. वर्ष 2019-20 में प्राकृतिक कृषि पद्धतियों को बढ़ावा देने हेतु PKVY की कौन-सी उप-योजना शुरू की गई थी?

(A) नमामि गंगे कार्यक्रम
(B) पूर्वोत्तर क्षेत्र हेतु जैविक मूल्य शृंखला विकास मिशन (MOVCDNER)
(C) परंपरागत कृषि विकास योजना (PKVY)
(D) भारतीय प्राकृतिक कृषि पद्धति (BPKP)

उत्तर: D

व्याख्या: 

  • भारतीय प्राकृतिक कृषि पद्धति (BPKP):
    • भारतीय प्राकृतिक कृषि पद्धति (BPKP) PKVY की एक उप-योजना है जिसका उद्देश्य प्राकृतिक कृषि पद्धतियों जैसे सिंथेटिक उर्वरकों और कीटनाशकों का उपयोग न करना, फसल सुरक्षा हेतु स्थानीय रूप से उपलब्ध संसाधनों का उपयोग एवं पर्यावरण के अनुकूल तकनीकों को अपनाना है।
      • यह योजना क्लस्टर-आधारित दृष्टिकोण के माध्यम से कार्यान्वित की जाती है तथा प्राकृतिक कृषि पद्धतियों को अपनाने हेतु किसानों के क्षमता निर्माण पर केंद्रित है।
    • BPKP के अंतर्गत किसानों को वर्मी कम्पोस्ट इकाइयों की स्थापना, गो-वंश- आधारित कृषि प्रणालियों, स्वदेशी आदानों की तैयारी आदि के लिये वित्तीय सहायता प्रदान की जाती है। यह योजना प्राकृतिक कृषि पद्धतियों को बढ़ावा देने हेतु प्रदर्शन भूखंडों के एक नेटवर्क की स्थापना का भी प्रावधान करती है।
      • यह टिकाऊ कृषि की दिशा में एक महत्त्वपूर्ण पहल है तथा इसका उद्देश्य प्राकृतिक संसाधनों एवं जैव विविधता के संरक्षण को बढ़ावा देना है। इससे किसानों को मृदा स्वास्थ्य में सुधार, फसल उत्पादकता बढ़ाने, इनपुट लागत को कम करने और आय बढ़ाने में मदद मिलने की उम्मीद है। अतः विकल्प D सही है।

प्रश्न 54. सरकार द्वारा पहचाने गए बागवानी क्लस्टर्स के संबंध में निम्नलिखित कथनों पर विचार कीजिये:

  1. सरकार ने 55 बागवानी क्लस्टर्स की पहचान की है।
  2. क्लस्टर विकास कार्यक्रम (CDP) के प्रायोगिक चरण के लिये 12 क्लस्टर्स का चयन किया गया है।
  3. बागवानी क्लस्टर्स केवल फलों एवं सब्जियों को कवर करते हैं।

उपर्युक्त कथनों में से कौन-सा/से सही है/हैं?

(A) केवल 1 और 2 
(B) केवल 2
(C) केवल 3
(D) केवल 1

उत्तर: A

व्याख्या:

  • सरकार ने वास्तव में 55 बागवानी क्लस्टर्स की पहचान की है। अतः कथन 1 सही है।
  • जैसा कि आर्थिक सर्वेक्षण में उल्लेख किया गया है, क्लस्टर विकास कार्यक्रम (CDP) के प्रायोगिक चरण के लिये 12 क्लस्टरों का चयन किया गया है। अतः कथन 2 सही है।
  • आर्थिक सर्वेक्षण में कहा गया है कि, बागवानी को बढ़ावा देने की योजना में "फल, सब्जियाँ, जड़ एवं कंद फसलें, मसाले, पुष्प, वृक्षारोपण फसलें आदि शामिल हैं। अतः कथन 3 सही नहीं है।

प्रश्न 55. मोटे अनाज या ‘मिलेट्स’ (Millets) और उसके लाभों के संदर्भ में निम्नलिखित कथनों पर विचार कीजिये:

  1. मिलेट्स अपनी आजीविका उत्पन्न करने, किसानों की आय बढ़ाने तथा पूरे विश्व में खाद्य एवं पोषण सुरक्षा सुनिश्चित करने की क्षमता के कारण महत्त्वपूर्ण हैं।
  2. भारत 50.9 मिलियन टन से अधिक मिलेट्स का उत्पादन करता है जो एशिया के 80% और वैश्विक उत्पादन का 20% है।
  3. मिलेट्स मुख्य रूप से एक रबी फसल है जो अधिकतर सिंचित परिस्थितियों में उपजाई जाती है, जिसमें अन्य प्रमुख फसलों की तुलना में अधिक जल एवं कृषि आदानों की आवश्यकता होती है।
  4. मिलेट्स को भारत सरकार द्वारा पोषक अनाज के रूप में मान्यता नहीं दी गई है।

उपर्युक्त कथनों में से कौन-सा/से सही है/हैं?

(A) केवल 1 और 2 
(B) केवल 2 केवल 3 
(C) केवल 1, 2 और 4
(D) केवल 3 और 4

उत्तर: A

व्याख्या:

  • मोटे अनाज/ या ‘मिलेट्स’ (Millets):
    • मिलेट्स ने आजीविका उत्पन्न करने, किसानों की आय बढ़ाने और पुरे विश्व में खाद्य एवं पोषण सुरक्षा सुनिश्चित करने की अपनी क्षमता के कारण महत्त्व प्राप्त किया है। मिलेट्स उपजाना आसान है क्योंकि कम जल की आवश्यकता होती है तथा यह मिट्टी के प्रकारों एवं जलवायु परिस्थितियों की एक विस्तृत शृंखला के अनुकूल हो सकता है। इसके पास उच्च पोषण मूल्य भी है तथा ग्लुटिन (Gluten) मुक्त हैं जो उन्हें ग्लुटिन असहिष्णुता वाले लोगों हेतु एक स्वस्थ विकल्प बनाते हैं। अतः कथन 1 सही है।
    • भारत मिलेट्स का सबसे बड़ा उत्पादक है, जो एशिया के 80% और वैश्विक उत्पादन का 20% उत्पादित करता है। भारत में, मिलेट्स शुष्क भूमि क्षेत्रों में उपजाया जाता है तथा छोटे एवं सीमांत किसानों हेतु एक महत्त्वपूर्ण फसल है। मिलेट्स भारत में कई समुदायों के पारंपरिक आहार का एक अभिन्न हिस्सा भी है। अतः कथन 2 सही है।
    • मिलेट्स मुख्य रूप से वर्षा आधारित फसलें हैं, जो शुष्क भूमि क्षेत्रों में बहुत कम या बिना सिंचाई के उपजी जाती हैं। उन्हें चावल एवं गेहूँ जैसी अन्य मुख्य फसलों की तुलना में कम जल और कृषि आदानों की आवश्यकता होती है, जिससे वे किसानों हेतु अधिक टिकाऊ विकल्प बन जाते हैं। अतः कथन 3 सही नहीं है।
    • मिलेट्स को भारत सरकार द्वारा पोषक अनाज के रूप में मान्यता दी गई है। वास्तव में, भारत सरकार ने मिलेट्स की कृषि एवं खपत को बढ़ावा देने हेतु राष्ट्रीय पोषक अनाज मिशन नामक एक योजना शुरू की है।  
      • सरकार ने लोगों, विशेषकर बच्चों एवं महिलाओं के पोषण की स्थिति में सुधार के लिये सार्वजनिक वितरण प्रणाली और मध्याह्न भोजन योजना में मोटे अनाज को भी शामिल किया है। अतः कथन 4 सही नहीं है।

प्रश्न 56. निम्नलिखित में से कौन-सा भारत में खाद्य प्रसंस्करण क्षेत्र के विकास हेतु आपूर्ति-पक्ष बाधा है?

(A) पर्याप्त और कुशल कोल्ड चेन इंफ्रास्ट्रक्चर का अभाव।
(B) कोल्ड स्टोरेज इंफ्रास्ट्रक्चर का असमान भौगोलिक वितरण।
(C) कनेक्टिविटी से संबंधित लॉजिस्टिक बाधाएँ।
(D) उपभोक्ता बास्केट में प्रसंस्कृत खाद्य पदार्थों की मांग में कमी।

उत्तर: A

व्याख्या:

  • भारत में खाद्य प्रसंस्करण उद्योग तेज़ी से वृद्धि कर रहा है, लेकिन पर्याप्त एवं कुशल कोल्ड चेन इंफ्रास्ट्रक्चर की कमी इसके विकास के लिये एक प्रमुख आपूर्ति-पक्ष बाधा है। कोल्ड चेन इंफ्रास्ट्रक्चर में भंडारण, परिवहन और वितरण सुविधाएँ शामिल हैं जो फलों, सब्जियों, डेयरी और मांस उत्पादों जैसे खराब होने वाले खाद्य उत्पादों की गुणवत्ता एवं सुरक्षा को बनाए रखने हेतु आवश्यक हैं।
  • भारत में खराब होने वाली उपज का एक बड़ा हिस्सा अपर्याप्त कोल्ड स्टोरेज सुविधाओं और परिवहन प्रणालियों के कारण नष्ट हो जाता है। मौजूदा कोल्ड स्टोरेज का बुनियादी ढाँचा असमान रूप से वितरित है, जिसमें अधिकांश सुविधाएँ कुछ राज्यों और शहरों में केंद्रित हैं, जिससे किसानों एवं खाद्य प्रसंस्करणकर्त्ताओं हेतु उन तक पहुँच बनाना मुश्किल हो जाता है। इसके अलावा, मौजूदा कोल्ड स्टोरेज के बुनियादी ढाँचे की गुणवत्ता अक्सर खराब होती है, जिसके परिणामस्वरूप उत्पाद खराब और बर्बाद हो जाता है।
  • कुशल कनेक्टिविटी और लॉजिस्टिक बाधाओं की कमी भी खाद्य प्रसंस्करण क्षेत्र के विकास के लिये चुनौती पेश करती है। उदाहरण के लिये, खराब सड़क अवसंरचना और अपर्याप्त परिवहन सुविधाएँ खेत से प्रसंस्करण सुविधा या बाज़ार तक खराब होने वाले उत्पादों के परिवहन के समय और लागत को बढ़ा सकती हैं, जिससे उत्पादों की गुणवत्ता एवं निधानी आयु (शेल्फ-लाइफ) कम हो जाती है।
  • हालाँकि, उपभोक्ता बास्केट में प्रसंस्कृत खाद्य पदार्थों की मांग में कमी आपूर्ति पक्ष की बाधा नहीं है। बल्कि, यह एक मांग-पक्ष की बाधा है जिसे प्रभावी विपणन और उपभोक्ता शिक्षा पहलों के माध्यम से दूर किया जा सकता है। अतः विकल्प A सही है।

प्रश्न 57. रबी विपणन मौसम (RMS) 2022-23 के दौरान RMS 2021-22 की तुलना में गेहूँ की कम खरीद का क्या कारण था? 

(A) रबी विपणन मौसम 2022-23 में गेहूँ खरीद का अनुमानित लक्ष्य कम था। 
(B) गेहूँ का बाज़ार मूल्य उसके खरीद सत्र के दौरान उसके न्यूनतम समर्थन मूल्य (MSP) से अधिक था। 
(C) मौसम की स्थिति के कारण गेहूँ की कमी थी। 
(D) उपरोक्त सभी।

उत्तर: B

व्याख्या:

  • रबी विपणन मौसम (RMS) 2021-22 की तुलना में रबी विपणन मौसम (RMS) 2022-23 के दौरान गेहूँ की कम खरीद का कारण यह था कि गेहूँ का बाज़ार मूल्य इसके खरीद मौसम के दौरान न्यूनतम समर्थन मूल्य (MSP) से अधिक था। अतः विकल्प B सही है।
    • न्यूनतम समर्थन मूल्य (MSP) वह मूल्य है जिस पर सरकार किसानों से फसल खरीदती है। जब किसी फसल का बाज़ार मूल्य उसके MSP से कम होता है, तो सरकार किसानों का समर्थन करने एवं खाद्य सुरक्षा सुनिश्चित करने हेतु MSP पर फसल खरीदती है। 
    • हालाँकि, जब किसी फसल का बाज़ार मूल्य उसके MSP से अधिक होता है, तो किसान अपनी फसलों को MSP पर सरकार को बेचने के बज़ाय खुले बाज़ार में बेचना पसंद करते हैं। इससे सरकार द्वारा खरीद कम हो जाती है।
    • रबी विपणन मौसम (RMS) 2022-23 के दौरान गेहूँ के मामले में, गेहूँ का बाज़ार मूल्य MSP से अधिक था, जिसके कारण सरकार द्वारा कम खरीद की गई।

प्रश्न 58. भारत के राष्ट्रीय स्तर पर निर्धारित अद्यतन योगदान (NDC) के संबंध में निम्नलिखित कथनों पर विचार कीजिये:

  1. भारत का लक्ष्य वर्ष 2005 के स्तर से वर्ष 2030 तक अपने सकल घरेलू उत्पाद की उत्सर्जन की मात्रा को 45% तक कम करना है।
  2. वर्ष 2030 तक गैर-जीवाश्म ईंधन-आधारित ऊर्जा संसाधनों से लगभग 50 प्रतिशत संचयी विद्युत उर्जा की स्थापित क्षमता प्राप्त करना है
  3. वर्ष 2030 तक अतिरिक्त वन और वृक्षों के आवरण के माध्यम से 2.5 से 5 बिलियन टन CO2 के अतिरिक्त कार्बन सिंक का निर्माण करना है। 

उपर्युक्त कथनों में से कौन-सा/से सही है/हैं?

(A) केवल 1 और 2 
(B) केवल 2 और 3 
(C) केवल 1 और 3
(D) 1, 2 और 3 

उत्तर: A 

व्याख्या: 

राष्ट्रीय स्तर पर निर्धारित योगदान (NDC)

  • भारत ने अपना पहला NDC अक्तूबर 2015 में UNFCCC को प्रस्तुत किया था। इसे अगस्त 2022 में  अद्यतन किया गया था।
    • वर्ष 2015 के NDC में आठ लक्ष्य शामिल थे, इनमें से तीन वर्ष 2030 तक प्राप्त किये जाने वाले मात्रात्मक लक्ष्य हैं।
    • तीन लक्ष्यों में गैर-जीवाश्म स्रोतों से  संचयी विद्युत ऊर्जा संस्थापित क्षमता 40% तक पहुँचना, वर्ष 2005 के स्तरों की तुलना में सकल घरेलू उत्पाद की उत्सर्जन की मात्रा 33 से 35% की कमी और अतिरिक्त वन और वृक्ष आच्छादन के माध्यम से 2.5 से 3 बिलियन टन CO2 के समतुल्य अतिरिक्त कार्बन सिंक का निर्माण करना शामिल है। 
  • पेरिस समझौते के अनुच्छेद 4 में यह प्रावधान किया गया है कि प्रत्येक पक्षकार प्रत्येक पाँच वर्ष में अपने NDC को संप्रेषित या अद्यतन करेगा।
  • भारत के संशोधित NDC हैं:
    • वर्ष 2005 के स्तर से वर्ष 2030 तक अपने सकल घरेलू उत्पाद की उत्सर्जन की मात्रा को 45% तक कम करना। अतः कथन 1 सही है।
    • हरित जलवायु कोष (Green Climate Fund- GCF) सहित प्रौद्योगिकी के हस्तांतरण और कम लागत वाले अंतर्राष्ट्रीय वित्त के की मदद से वर्ष 2030 तक गैर-जीवाश्म ईंधन-आधारित ऊर्जा संसाधनों से लगभग 50% संचयी विद्युत ऊर्जा स्थापित क्षमता प्राप्त करना है। अतः कथन 2 सही है।
    • वर्ष 2030 तक अतिरिक्त वन और वृक्षों के आवरण के माध्यम से 2.5 से 3 बिलियन टन CO2 के बराबर अतिरिक्त कार्बन सिंक का निर्माण करना है। अतः कथन 3 सही नहीं है।

प्रश्न 59. भारत वन स्थिति रिपोर्ट (ISFR) 2021 के संदर्भ में निम्नलिखित कथनों पर विचार कीजिये:

  1. भारतीय राज्यों के मध्य, वनों में सर्वाधिक कार्बन स्टॉक मध्य प्रदेश में दर्ज किया।
  2. वर्ष 2013 की तुलना में वर्ष 2021 में देश के मैंग्रोव कवर में वृद्धि देखी गई।

उपर्युक्त कथनों में से कौन-सा/से सही है/हैं?

(A) केवल 1
(B) केवल 2
(C) 1 और 2 दोनों
(D) न तो 1 न ही 2 

उत्तर: B 

व्याख्या: 

भारत वन स्थिति रिपोर्ट (ISFR) 2021

  • भारतीय वन सर्वेक्षण (Forest Survey of India- FSI) ने वर्ष 2004 में पहली बार और वर्ष 2011 से द्विवार्षिक रूप से भारत के वनों में कार्बन स्टॉक (जो कि कार्बन की वह मात्रा है जो वायुमंडल से पृथक होती है एवं बायोमास, डेडवुड, मृदा और वन्य अपशिष्ट के रूप में संग्रहीत होती है) का आकलन किया । 
    • भारत वन स्थिति रिपोर्ट (Indian State of Forest Report - ISFR) का अनुमान है कि वर्ष 2019 में वनों और वृक्षों के आवरण का कार्बन स्टॉक लगभग 7,204 मिलियन टन का अनुमानित है, जो कि वर्ष 2017 में किये गये  पिछले आकलन के अनुमानों की तुलना में 79.4 मिलियन टन कार्बन स्टॉक से अधिक है।
    • यह वन और वृक्षों के आवरण के माध्यम से हुए कार्बन उत्सर्जन की मात्रा 30.1 बिलियन टन कार्बन डाइऑक्साइड (CO2) के समतुल्य होने के लिये परिवर्तित होता है।
    • भारतीय राज्यों में अरुणाचल प्रदेश के वनों में अधिकतम कार्बन स्टॉक (1023.84 मिलियन टन) है, इसके बाद मध्य प्रदेश (609.25 मिलियन टन) का स्थान है। अतः कथन 1 सही नहीं है।
    • विभिन्न राज्यों/केंद्रशासित प्रदेशों में प्रति हेक्टेयर वन कार्बन स्टॉक यह निर्दिष्ट करता है कि जम्मू और कश्मीर 173.41 टन प्रति हेक्टेयर कार्बन स्टॉक का अधिकतम योगदान दे रहा है, इसके बाद हिमाचल प्रदेश (167.0 टन), सिक्किम (166.2 टन) तथा अंडमान और निकोबार द्वीप समूह (162.9 टन) का स्थान है।
  • ISFR 2021 के अनुसार, वर्ष 2013 की तुलना में वर्ष 2021 में देश में मैंग्रोव कवर में 364 वर्ग किमी की वृद्धि हुई है। अतः कथन 2 सही है।

प्रश्न 60. रामसर स्थल  के संबंध में निम्नलिखित कथनों पर विचार कीजिये: 

  1. यह यूनेस्को द्वारा स्थापित रामसर कन्वेंशन, 1971 के तहत अंतर्राष्ट्रीय महत्त्व की एक आर्द्रभूमि है और इसका नाम ईरान के रामसर शहर के नाम पर रखा गया है।
  2. पश्चिम बंगाल में सुंदरबन भारत का सबसे बड़ा रामसर स्थल है।

उपर्युक्त कथनों में से कौन-सा/से सही नहीं है/हैं?

(A) केवल 1
(B) केवल 2
(C) 1 और 2 दोनों
(D) न तो 1  न ही 2 

उत्तर: D 

व्याख्या: 

रामसर साइट 

  • रामसर स्थल रामसर सम्मेलन के अंतर्गत अंतर्राष्ट्रीय महत्त्व की एक आर्द्रभूमि है, जिसे वर्ष 1971 में यूनेस्को द्वारा स्थापित एक अंतर-सरकारी पर्यावरण संधि 'वेटलैंड्स पर कन्वेंशन' के रूप में भी जाना जाता है और इसका नाम ईरान के रामसर शहर के नाम पर रखा गया है, जहाँ उस वर्ष सम्मेलन पर हस्ताक्षर किये गए थे। अतः कथन 1 सही है।
  • रामसर मान्यता दुनिया भर में आर्द्रभूमि की पहचान है जो अंतर्राष्ट्रीय महत्त्व के हैं, खासकर अगर वे जलपक्षी (पक्षियों की लगभग 180 प्रजातियों) को आवास प्रदान करते हैं।
  • ऐसी आर्द्रभूमियों के संरक्षण और उनके संसाधनों के विवेकपूर्ण उपयोग में अंतर्राष्ट्रीय हित और सहयोग शामिल है।
  • पश्चिम बंगाल में सुंदरबन भारत का सबसे बड़ा रामसर स्थल है। अतः कथन 2 सही है।

प्रश्न 61. नमामि गंगे कार्यक्रम के संबंध में निम्नलिखित कथनों पर विचार कीजिये:

  1. राष्ट्रीय स्वच्छ गंगा मिशन (NMCG) इस कार्यक्रम के कार्यान्वयन के लिये ज़िम्मेदार संस्था है। 
  2. इसका नोडल मंत्रालय, जल शक्ति मंत्रालय है।
  3. कार्यक्रम के दूसरे चरण में परियोजनाओं को पूरा करने और विश्वसनीय विस्तृत परियोजना रिपोर्ट (DPR) तैयार करने पर ध्यान केंद्रित किया जाएगा। 

उपर्युक्त कथनों में से कौन-सा/से सही है/हैं?

(A) केवल 1 और 2
(B) केवल 2 और 3 
(C) केवल 1 और 3
(D) 1, 2 और 3 

उत्तर: D 

व्याख्या: 

नमामि गंगे कार्यक्रम 

  • यह एक एकीकृत संरक्षण मिशन है, जिसे जून 2014 में केंद्र सरकार द्वारा ‘फ्लैगशिप कार्यक्रम' के रूप में अनुमोदित किया गया था, ताकि प्रदूषण के प्रभावी उन्मूलन और राष्ट्रीय नदी गंगा के संरक्षण एवं कायाकल्प के दोहरे उद्देश्यों को पूरा किया जा सके। 
  • इसे  जल संसाधन मंत्रालय, नदी विकास और गंगा संरक्षण विभाग तथा जल शक्ति मंत्रालय के अंतर्गत संचालित किया जा रहा है। अतः कथन 2 सही है।
  • यह कार्यक्रम राष्ट्रीय स्वच्छ गंगा मिशन (NMCG) और इसके राज्य समकक्ष संगठनों यानी राज्य कार्यक्रम प्रबंधन समूहों (SPMG) द्वारा कार्यान्वित किया जा रहा है। अतः कथन 1 सही है।
  • नमामि गंगे कार्यक्रम (2021-26) के दूसरे चरण में राज्य परियोजनाओं को तेज़ी से पूरा करने और गंगा के सहायक शहरों में परियोजनाओं के लिये विश्वसनीय विस्तृत परियोजना रिपोर्ट (Detailed Project Report- DPR) तैयार करने पर ध्यान केंद्रित करेंगे। अतः कथन 3 सही है।
    • छोटी नदियों और आर्द्रभूमि के पुनरुद्धार पर भी ध्यान दिया जा रहा है।
    • प्रत्येक प्रस्तावित गंगा ज़िले में कम से कम 10 आर्द्रभूमि हेतु वैज्ञानिक योजना और स्वास्थ्य कार्ड विकसित करना है और उपचारित जल एवं अन्य उत्पादों के पुन: उपयोग के लिये नीतियों को अपनाना है।

प्रश्न 62. "पीएम गतिशक्ति" के संबंध में निम्नलिखित कथनों पर विचार कीजिये:

  1. पीएम गतिशक्ति बहुविध कनेक्टिविटी के लिये एक राष्ट्रीय मास्टर प्लान है।
  2. यह नियोजन और कार्यान्वयन के लिये विभिन्न मंत्रालयों को एक साथ लाने का एक डिजिटल प्लेटफॉर्म है।

उपर्युक्त कथनों में से कौन-सा/से सही है/हैं?

(A) केवल 1
(B) केवल 2
(C) 1 और 2 दोनों
(D) न तो 1 न ही 2

उत्तर: C

व्याख्या:

  • पीएम गतिशक्ति जिसे बहुविध कनेक्टिविटी के लिये राष्ट्रीय मास्टर प्लान के रूप में भी जाना जाता है, इसे पिछले वर्ष पेश किया गया था। अतः कथन 1 सही है।
  • राष्ट्रीय रसद नीति पीएम गतिशक्ति पहल का समर्थन करेगी और इसकी पूरक होगी।
  • यह इन्फास्ट्रक्चर कनेक्टिविटी परियोजनाओं की एकीकृत नियोजन और समन्वित कार्यान्वयन के लिये रेलवे और सड़क सहित 16 मंत्रालयों को एक साथ लाने वाला एक डिजिटल प्लेटफॉर्म है। अतः कथन 2 सही है।
  • बहुविध कनेक्टिविटी परिवहन के एक माध्यम से दूसरे माध्यम में लोगों, वस्तुओं और सेवाओं की आवाजाही के लिये एकीकृत और निर्बाध कनेक्टिविटी प्रदान करेगी।

प्रश्न 63. निम्नलिखित में से कौन-सा/से कथन सही है/हैं?

  1. राष्ट्रीय मुद्रीकरण पाइपलाइन (NMP) भारत सरकार द्वारा सार्वजनिक अवसंरचना संपत्तियों के मुद्रीकरण के लिये एक नीतिगत पहल है।
  2. NMP रणनीतिक विनिवेश के भारत सरकार के एजेंडे के साथ जुड़ा हुआ है।

नीचे दिये गए कूट का प्रयोग कर सही उत्तर चुनिये:

(A) केवल 1
(B) केवल 2
(C) 1 और 2 दोनों
(D) न तो 1 न ही 2

उत्तर: C

व्याख्या:

  • राष्ट्रीय मुद्रीकरण पाइपलाइन (NMP) भारत सरकार द्वारा सार्वजनिक अवसंरचना संपत्तियों, जैसे सड़कों, पुलों, हवाई अड्डों और बंदरगाहों को विभिन्न वित्तीय साधनों जैसे लंबी अवधि के पट्टे, रियायतें और निर्माण-परिचालन-हस्तांतरण (Build-Operate-Transfer) मॉडल के माध्यम से मुद्रीकृत करने के लिये एक नीतिगत पहल है। अतः कथन 1 सही है।
  • NMP का उद्देश्य सार्वजनिक अवसंरचना संपत्तियों की संभावनाओं को सामने लाना है और उनकी दक्षता एवं प्रदर्शन को प्रोत्साहित करने के लिये अवसंरचना के विकास में निजी निवेश को बढ़ावा देना है।
  • राष्ट्रीय मुद्रीकरण पाइपलाइन (NMP) को भारत सरकार के रणनीतिक विनिवेश के एजेंडे के साथ जोड़ा गया है, जिसमें सार्वजनिक क्षेत्र के उद्यमों (PSE) या सार्वजनिक अवसंरचना संपत्तियों के स्वामित्व का आंशिक या पूर्ण हस्तांतरण के लिये निजी क्षेत्र को शामिल करता है। अतः कथन 2 सही है।

प्रश्न 64. 'उड़ान (उड़े देश का आम नागरिक)' योजना के संदर्भ में निम्नलिखित कथनों पर विचार कीजिये:

  1. इस योजना के अंतर्गत क्षेत्रीय विमान संपर्क निधि  (RCF) बनाई गई थी, जो योजना की व्यवहार्यता अंतराल निधि (VGF) आवश्यकताओं को कोष प्रदान करता है।
  2. यह योजना देश के टियर-2 और 3 शहरों के बीच कनेक्टिविटी पर केंद्रित है।
  3. यह अंतर्राष्ट्रीय उड्डयन बाज़ार के विकास को भी बढ़ावा दे रहा है।

उपर्युक्त कथनों में से कौन-सा/से सही है/हैं?

(A) केवल 1 और 2
(B) केवल 2 और 3
(C) 1 और 3
(D) 1, 2 और 3

उत्तर: A

व्याख्या:

  • उड़े देश का आम नागरिक (UDAN) को वर्ष 2016 में नागरिक उड्डयन मंत्रालय के अंतर्गत एक क्षेत्रीय संपर्क योजना के रूप में लॉन्च किया गया था।
  • इस योजना के अंतर्गत क्षेत्रीय विमान संपर्क निधि (RCF) बनाया गया था, जो कुछ घरेलू उड़ानों पर लेवी के माध्यम से योजना की व्यवहार्यता अंतराल निधि (VGF) आवश्यकताओं को पूरा करता है। अतः कथन 1 सही है।
  • यह योजना भारत में टियर -2 और टियर 3 शहरों के बीच कनेक्टिविटी पर केंद्रित है। अतः कथन 2 सही है।
  • इस योजना का उद्देश्य छोटे शहरों में भी आम आदमी को क्षेत्रीय मार्गों पर सस्ती, आर्थिक रूप से व्यवहार्य और लाभदायक हवाई यात्रा उपलब्ध कराना है।
  • इसका उद्देश्य क्षेत्रीय विमानन बाज़ार का विकास करना है। अतः कथन 3 सही नहीं है

प्रश्न 65. भारतीय रेलवे (IR) के संदर्भ में निम्नलिखित कथनों पर विचार कीजिये:

  1. भारतीय रेलवे (IR) दुनिया का चौथा सबसे बड़ा रेलवे नेटवर्क है।
  2. रेलवे के बुनियादी ढाँचे में पिछले चार वर्षों में कोविड-19 के कारण पूंजीगत व्यय में गिरावट आई है।

उपर्युक्त कथनों में से कौन-सा/से सही है/हैं?

(A) केवल 1
(B) केवल 2
(C) 1 और 2 दोनों
(D) न तो 1  न ही 2

उत्तर: A

व्याख्या:

  • भारतीय रेलवे (IR)  68,031 किलोमीटर से अधिक के विस्तृत मार्ग के साथ दुनिया का चौथा सबसे बड़ा रेलवे नेटवर्क है। अतः कथन 1 सही है।
  • भारतीय रेल द्वारा अवसंरचना वृद्धि की तीव्र गति धन के आवंटन में पर्याप्त वृद्धि और सरकार द्वारा उठाए जा रहे विभिन्न कदमों का परिणाम है।
  • रेलवे में बुनियादी ढाँचे पर पूंजीगत व्यय को वर्ष 2014 के बाद से अत्यधिक प्रोत्साहन मिला है। इसमें पिछले चार वर्षों में पूंजीगत व्यय में लगातार वृद्धि देखी गई है। वित्त वर्ष 2023 में 2.5 लाख करोड़ का पूंजीगत व्यय  पिछले वर्ष की तुलना में लगभग 29% अधिक है। अतः कथन 2 सही नहीं है।

प्रश्न 66. निम्नलिखित में से कौन-से क्षेत्र राष्ट्रीय अवसंरचना पाइपलाइन के अंतर्गत आते हैं?

(A) परिवहन, आवास, वाणिज्यिक विकास, दूरसंचार और स्वच्छता
(B) स्वास्थ्य सेवा, शिक्षा, कृषि, ऊर्जा और पर्यटन
(C) प्रौद्योगिकी, मनोरंजन, मीडिया, वित्त और विनिर्माण
(D) उपरोक्त में से कोई नहीं

उत्तर: A

व्याख्या:

  • राष्ट्रीय अवसंरचना पाइपलाइन (NIP) भारत में अवसंरचना के विकास के लिये एक अग्रगामी कार्यक्रम संबंधी दृष्टिकोण है, जिसे 2019 में भारत सरकार द्वारा अपनाया गया है।
  • परिवहन, आवास, वाणिज्यिक विकास, दूरसंचार और स्वच्छता, राष्ट्रीय अवसंरचना पाइपलाइन के अंतर्गत आने वाले क्षेत्र हैं। अतः विकल्प A सही है।
  • इसका उद्देश्य अवसंरचना के विकास हेतु  विभिन्न क्षेत्रों को एक साथ लाना और इन सभी क्षेत्रों के लिये समन्वित और एकीकृत रूप से कार्यान्वयन सुनिश्चित करना है साथ ही अवसंरचना के विकास को सुविधाजनक बनाते हुए देश के सामाजिक-आर्थिक विकास को प्रोत्साहन देना है।

प्रश्न 67. 'भारत की अवसंरचनात्मक यात्रा' के संदर्भ में निम्नलिखित कथनों पर विचार कीजिये:

  1. भारत की अवसंरचनात्मक यात्रा दृष्टिकोण में वैश्विक रही है, लेकिन कार्यान्वयन में राष्ट्रीय रही है।
  2. सार्वजनिक क्षेत्र ने भारत के डिजिटल अवसंरचना के विकास में कोई महत्त्वपूर्ण भूमिका नहीं निभायी है।
  3. भारत ने पिछले कुछ वर्षों में केवल भौतिक अवसंरचना के विकास पर ध्यान केंद्रित किया है।

उपर्युक्त कथनों में से कौन-सा/से सही है/ हैं?

(A) केवल 1
(B) केवल 2 और 3
(C) केवल 3
(D) 1, 2 और 3

उत्तर: A

व्याख्या:

  • यह दर्शाती है कि भारत ने सड़कों, रेलवे, जलमार्गों, बंदरगाहों और हवाई अड्डों आदि की संख्या में अभूतपूर्व विस्तार हुआ है, जिससे देश को यूनिमॉडल से मल्टी-मॉडल परिवहन की ओर बढ़ने में मदद मिली है।
  • भारत की अवसंरचनात्मक यात्रा दृष्टिकोण में वैश्विक रही है, लेकिन नवाचार और कार्यान्वयन में राष्ट्रीय रही है। देश ने सार्वजनिक क्षेत्र के माध्यम से प्रौद्योगिकी और डिजिटल कनेक्टिविटी में नवाचार का नेतृत्त्व किया है। अतः कथन 1 सही है।
  • भारत में अवसंरचना का विकास इस तथ्य का प्रमाण है कि भारत उन कुछ देशों में से एक रहा है जहाँ सार्वजनिक क्षेत्र द्वारा प्रौद्योगिकी और डिजिटल कनेक्टिविटी में नवाचार किया गया है और आगे भी किया जा रहा है। अतः कथन 2 सही नहीं है।
  • भारत की अवसंरचनात्मक यात्रा भौतिक और डिजिटल दोनों अवसंरचनाओं के विकास को कवर करती है। अतः कथन 3 सही नहीं है।

प्रश्न 68. निम्नलिखित कथनों पर विचार कीजिये:

  1. नेशनल पेमेंट्स कॉरपोरेशन ऑफ इंडिया इंटरनेशनल (NPCIL) के नेतृत्व में यूनिफाइड पेमेंट इंटरफेस (UPI) ने भारत में भुगतान परिदृश्य को बदल दिया है।
  2. डिजिटल परिवर्तन ने व्यापार प्रक्रियाओं और संगठनात्मक संरचनाओं के पुनर्विचार को सक्षम बनाया है।

उपर्युक्त कथनों में से कौन-सा/से सही है/हैं?

(A) केवल 1
(B) केवल 2
(C) 1 और 2 दोनों
(D) न तो 1 न ही 2

उत्तर: C

व्याख्या:

  • नेशनल पेमेंट्स कॉरपोरेशन ऑफ इंडिया इंटरनेशनल (NPCIL) द्वारा यूनिफाइड पेमेंट इंटरफेस (UPI) की शुरुआत से भारत में भुगतान परिदृश्य में एक महत्त्वपूर्ण परिवर्तन आया है। अतः कथन 1 सही है।
  • नवोन्मेष को प्रोत्साहन देने के पीछे कई कारक हो सकते हैं, जिसमें प्रौद्योगिकी में प्रगति, उपभोक्ता वरीयताओं में परिवर्तन और बाज़ार की स्थितियों में परवर्तन आदि शामिल हैं।
  • डिजिटल परिवर्तन में न केवल नई तकनीकों को अपनाना शामिल है बल्कि व्यावसायिक प्रक्रियाओं और संगठनात्मक संरचनाओं पर पुनर्विचार भी शामिल है। अतः कथन 2 सही है।

प्रश्न 69. निम्नलिखित में से कौन-सा/से कथन सही नहीं है/हैं?

  1. GST सहाय प्लेटफॉर्म GSTN द्वारा, करदाताओं को GST से संबंधित उनकी चिंताओं और प्रश्नों को दूर करने में सहायता करने के लिये, बनाया गया था।
  2. GSTN को एक गैर-लाभकारी, सार्वजनिक-निजी भागीदारी कंपनी के रूप में स्थापित किया गया था।

नीचे दिये गए कूट का प्रयोग कर सही उत्तर चुनिये:

(A) केवल 1
(B) केवल 2
(C) 1 और 2 दोनों
(D) न तो 1 न ही 2

उत्तर: D

व्याख्या:

  • GST सहाय भारत में वस्तु एवं सेवा कर नेटवर्क (GSTN) द्वारा विकसित एक डिजिटल प्लेटफॉर्म है, जो करदाताओं को वस्तु एवं सेवा कर (GST) से संबंधित उनके प्रश्नों और शिकायतों को हल करने में सहायता प्रदान करता है। अतः कथन 1 सही है।
  • GSTN को एक गैर-लाभकारी, सार्वजनिक-निजी भागीदारी कंपनी के रूप में स्थापित किया गया था, जिसका उद्देश्य केंद्र और राज्य सरकारों तथा करदाताओं एवं GST प्रणाली में अन्य हितधारकों को एक साझा IT अवसंरचना और सेवाएँ प्रदान करना था। अतः कथन 2 सही है।

प्रश्न 70. निम्नलिखित में से कौन-सा अर्थव्यवस्था पर बढ़े हुए अवसंरचनात्मक निवेश के प्रभाव को दर्शाता है?

  1. यह आर्थिक विकास में गिरावट का कारण बन सकता है।
  2. यह अर्थव्यवस्था की संभावित वृद्धि को एक महत्वपूर्ण गति प्रदान कर सकता है।

उपर्युक्त कथनों में से कौन-सा/से सही है/हैं?

(A) केवल 1
(B) केवल 2
(C) 1 और 2 दोनों
(D) न तो 1 न ही 2

उत्तर : B

व्याख्या:

  • अवसंरचनात्मक निवेश में वृद्धि से  आर्थिक विकास में गिरावट के स्थान पर वृद्धि होती है। अवसंरचना किसी भी अर्थव्यवस्था की रीढ़ होती है और इसके विकास से कई सकारात्मक आर्थिक प्रभाव हो सकते हैं। अतः कथन 1 सही नहीं है।
  • सड़कें, पुल और सार्वजनिक परिवहन प्रणाली जैसी अवसंरचनाएँ व्यवसायों के लिये रसद की ढुलाई और श्रमिकों की उनकी नौकरी तक पहुँच आसान और तीव्र बना सकती है। इससे उत्पादकता और दक्षता में वृद्धि हो सकती है, जिससे उच्च आर्थिक विकास को बढ़ावा मिल सकेगा। अतः कथन 2 सही है 

प्रश्न 71. निम्नलिखित कथनों पर विचार कीजिये:

  1. वर्ष 2021 में अमेरिका और जापान के बाद चीन विश्व का सबसे बड़ा बीमा बाज़ार बनकर उभरा।
  2. वर्ष 2021 में, भारत का जीवन बीमा निवेश अन्य उभरते बाज़ारों की तुलना में लगभग दोगुना था जो वैश्विक औसत से थोडा अधिक थी।

उपर्युक्त कथनों में से कौन-सा/से सही है/हैं?

(A) केवल 1
(B) केवल 2
(C) 1 और 2 दोनों
(D) न तो 1 और न ही 2

उत्तर : B 

व्याख्या : 

  • आर्थिक सर्वेक्षण वर्ष 2022-23 के अनुसार आने वाले दशक में भारतीय बीमा बाज़ार सबसे तेजी से उभरते बीमा बाज़ारों में से एक बनकर उभरेगा। यह बचत से जुड़े जीवन बीमा के ग्राहकों से संबंधित नुकसानों को भी दर्शाता है।
  • भारत में विक्रय किये जाने वाले अधिकांश जीवन बीमा उत्पाद केवल एक छोटी सी सुरक्षा बचत से संबंधित होते हैं। जिसके परिणामस्वरूप कमाने वाले व्यक्ति की समय से पूर्व मृत्यु होने की स्थिति में परिवारों को एक महत्त्वपूर्ण वित्तीय अंतर का सामना करना पड़ता है।
  • आर्थिक सर्वेक्षण दर्शाता कि कैसे सहस्राब्दी के अंत में देश के बीमा क्षेत्र में निवेश 2.7% से बढ़कर वर्ष 2020 में 4.2% हो गया। वर्ष 2021 में जीवन बीमा निवेश औसतन 3.2% था, जो उभरते बाज़ारों की तुलना में लगभग दोगुना था एवं वैश्विक स्तर से थोड़ा अधिक था। अतः कथन 2 सही है।
  • जीवन बीमा क्षेत्र में वैश्विक प्रीमियम वृद्धि ने 4.5% की वृद्धि दर्ज़ करते हुए मज़बूती से वापसी की। वर्ष 2021 में 2.8 ट्रिलियन USD के कुल प्रीमियम (गैर-जीवन और जीवन) के साथ अमेरिका विश्व का सबसे बड़ा बीमा बाज़ार बना रहा, इसके बाद चीन और जापान का स्थान रहा। अतः कथन 1 सही नहीं है।

प्रश्न 72. वित्तीय वर्ष 2023 के अनुसार सरकारी प्रतिभूतियों (G-Sec) की व्यापार की मात्रा के संदर्भ में निम्नलिखित कथनों पर विचार कीजिये:

  1. द्वितीयक बाज़ार में विदेशी बैंक उच्चतम व्यापारिक खंड के रूप में उभरे।
  2. निवल आधार पर, विदेशी बैंक और प्राथमिक व्यापारी शुद्ध विक्रेता थे।

उपर्युक्त कथनों में से कौन-सा/से सही है/हैं?

(A) केवल 1
(B) केवल 2
(C) 1 और 2 दोनों
(D) न तो 1 और न ही 2

उत्तर: B 

व्याख्या: 

  • सर्वेक्षण दस्तावेज़ के अनुसार, निजी क्षेत्र के बैंक वित्त वर्ष 2023 की दूसरी तिमाही के दौरान द्वितीयक बाज़ार में उच्चतम व्यापारिक खंड के रूप में उभरे, जिसमें शुद्ध एकमुश्त व्यापारिक गतिविधि के रूप में “क्रय” व्यापार में 25.0 प्रतिशत और “विक्रय” व्यापार में 24.8% की हिस्सेदारी थी। इसके बाद विदेशी बैंकों, सार्वजनिक क्षेत्र के बैंकों, प्राथमिक व्यापारियों और म्युचुअल फंड का स्थान रहा। अतः कथन 1 सही नहीं है।
  • निवल आधार पर, विदेशी बैंक और प्राथमिक व्यापारी शुद्ध विक्रेता थे। इसके विपरीत सार्वजनिक क्षेत्र के बैंक, सहकारी बैंक, वित्तीय संस्थान, बीमा कंपनियाँ,, म्युचुअल फंड, निजी क्षेत्र के बैंक और 'अन्य' द्वितीयक बाज़ार में शुद्ध खरीदार थे। अतः कथन 2 सही है।

प्रश्न 73. निम्नलिखित कथनों पर विचार कीजिये:

  1. अनुसूचित वाणिज्यिक बैंकों (SCB) का सकल गैर-निष्पादित आस्ति (GNPA) वर्ष 2022 में सात वर्ष के निचले पायदान पर आ गया है।
  2. दिवाला और शोधन अक्षमता संहिता (IBC) के माध्यम से अनुसूचित वाणिज्यिक बैंकों (SCB) की उगाही दर अन्य समूहों की तुलना में वित्त वर्ष 2022 में सबसे अधिक थी।

उपर्युक्त कथनों में से कौन-सा/से सही है/हैं?

(A) केवल 1
(B) केवल 2
(C) 1 और 2 दोनों
(D) न तो 1 न ही 2

उत्तर: C 

व्याख्या: 

  • आर्थिक सर्वेक्षण 2023 में यह उल्लेख किया गया है कि अनुसूचित वाणिज्यिक बैंकों (SCB) का सकल गैर-निष्पादित आस्ति (GNPA) वर्ष 2022 में सात वर्ष के निचले पायदान 5.0% पर आ गया है। अतः कथन 1 सही है।
  • सर्वेक्षण में कहा गया है कि पूंजी-से-जोखिम भारित आस्तिअनुपात (CRAR) 16.0 पर आ गया है जो 11.5 की विनियामक माँगों से काफी ऊपर है। सर्वेक्षण में यह भी कहा गया है कि NBFC के स्तर में भी सुधार जारी है।
  • दिवाला और शोधन अक्षमता संहिता (IBC) के माध्यम से अनुसूचित वाणिज्यिक बैंकों (SCB) की उगाही दर अन्य समूहों की तुलना में वित्त वर्ष 2022 में सबसे अधिक थी। अतः कथन 2 सही है।
  • मार्च 2020 में GNPA अनुपात 8.2% से घटकर सितंबर 2022 में 5.0% पर सात वर्ष के निचले पायदान पर आ गया, जबकि शुद्ध गैर-निष्पादनकारी परिसंपत्तियाँ (NNPA) कुल संपत्ति के 1.3% पर दस वर्ष के निचले पायदान पर आ गई हैं, सर्वेक्षण में उल्लेख किया गया है न्यूनतम स्लिपेज और बकाया GNPA में उगाही, पदोन्नत और बट्टे खातों में कमी के कारण यह गिरावट आई है।

प्रश्न 74. वर्ष 2022-23 में विदेशी पोर्टफोलियो निवेशकों (FPI) के प्रदर्शन के बारे में निम्नलिखित कथनों पर विचार कीजिये:

  1. वैश्विक कारकों द्वारा संचालित बहिर्वाह के बावजूद अवधारकों (होल्डिंग) के शुद्ध बाज़ार मूल्य को दर्शाते हुए FPI की अभिरक्षीय अवधारकों (कस्टोडियल होल्डिंग) में वृद्धि देखी गई है।
  2. घरेलू संस्थागत निवेशकों (DII) द्वारा निवेश के कारण बड़े पैमाने पर सुधार के लिये भारतीय इक्विटी बाज़ार अपेक्षाकृत कम संवेदनशील रहे हैं।

उपर्युक्त कथनों में से कौन-सा/से सही है/हैं?

(A) केवल 1
(B) केवल 2
(C) 1 और 2 दोनों
(D) न तो 1 और न ही 2

उत्तर: C 

व्याख्या: 

  • आर्थिक सर्वेक्षण 2022-2023 में इस बात पर प्रकाश डाला गया है कि भारतीय अर्थव्यवस्था के मज़बूत समष्टि अर्थव्यवस्था का आधार और समय-समय पर बाज़ार जोखिमों में सुधार के कारण भारत एक आकर्षक निवेश गंतव्य बना हुआ है।
  • अभिरक्षीय (कस्टोडियल) संपत्ति (अवधारकों के शुद्ध बाज़ार मूल्य को दर्शाते हुए FPI की अभिरक्षीय अवधारक) में वैश्विक कारकों द्वारा संचालित बहिर्वाह के बावजूद वृद्धि देखी गई। अतः कथन 1 सही है।
  • नवंबर 2021 की तुलना में नवंबर 2022 के अंत में FPI के पास शुद्ध अभिरक्षीय (कस्टोडियल) संपत्ति में 3.4% की वृद्धि हुई।
  • घरेलू संस्थागत निवेशकों (DII) द्वारा किये गए निवेश ने हाल के वर्षों में FPI बहिर्गमन के विपरीत प्रतिकारी बल के रूप में कार्य किया, जिससे भारतीय इक्विटी बाज़ार अपेक्षाकृत बड़े पैमाने पर सुधार के लिये कम संवेदनशील हो गया। अतः कथन 2 सही है।

प्रश्न 75. निम्नलिखित कथनों पर विचार कीजिये: 

  1. सितंबर 2022 तक, चल रही कॉरपोरेट इनसॉल्वेंसी रिज़ाॅल्यूशन प्रक्रिया (CIRP) की सबसे बड़ी संख्या सेवा क्षेत्र से संबंधित है।
  2. FY22 में, दिवाला एवं शोधन अक्षमता कोड (IBC) के तहत अनुसूचित वाणिज्यिक बैंकों द्वारा वसूल की गई कुल राशि सबसे अधिक रही है।

उपर्युक्त कथनों में से कौन-सा/से सही है/हैं?

(A) केवल 1
(B) केवल 2
(C) 1 और 2 दोनों
(D) न तो 1 और न ही 2

उत्तर: B 

व्याख्या: 

  • चल रही कॉरपोरेट इनसॉल्वेंसी रिज़ाॅल्यूशन प्रक्रिया (CIRP) का लगभग 52% उद्योग से संबंधित है, इसके बाद सितंबर वर्ष 2022 तक सेवा क्षेत्र में 37% है। अत: कथन 1 सही नहीं है।
  • RBI के आँकड़ों के अनुसार, FY22 में, दिवाला एवं शोधन अक्षमता कोड (Insolvency and Bankruptcy Code - IBC) के तहत अनुसूचित वाणिज्यिक बैंकों द्वारा वसूल की गई कुल राशि इस अवधि में लोक अदालतों, SARFAESI अधिनियम और DRT जैसे अन्य चैनलों की तुलना में सबसे अधिक रही है। अतः कथन 2 सही है।

प्रश्न 76. पूंजी बाज़ार में खुदरा भागीदारी के संदर्भ में निम्नलिखित कथनों पर विचार कीजिये:

  1. वित्त वर्ष 2012 के दौरान इसी अवधि की तुलना में वित्त वर्ष 2023 (अप्रैल-नवंबर 2022) के दौरान नकद सेगमेंट (खंड)में व्यक्तिगत निवेशकों की हिस्सेदारी में मामूली गिरावट आई है।
  2. FY22 के सापेक्ष FY23 के दौरान डीमैट खातों की वृद्धि में गिरावट आ रही है।

उपर्युक्त कथनों में से कौन-सा/से सही है/हैं?

(A) केवल 1
(B) केवल 2
(C) 1 और 2 दोनों
(D) न तो 1 न ही 2

उत्तर : C 

व्याख्या: 

  • वित्त वर्ष 2012 की अवधि की तुलना में वित्त वर्ष 2023 (अप्रैल-नवंबर 2022) के नकद सेगमेंट (खंड) में व्यक्तिगत निवेशकों की हिस्सेदारी में मामूली गिरावट आई है। अतः कथन 1 सही है।
  • हालाँकि, डीमैट खातों की संख्या में तीव्रता से वृद्धि हुई, जो नवंबर 2022 के अंत तक वर्ष दर वर्ष के आधार पर 39% अधिक थी।
  • वित्त वर्ष 2022 की तुलना में वित्त वर्ष 2023 में डीमैट खातों की संवर्द्धित वृद्धि के रुझान गिरते हुए नज़र आ रहे है, संभवत: द्वितीयक बाज़ार में बढ़ती अस्थिरता और चालू वित्त वर्ष के दौरान प्रचलित वैश्विक शीर्ष समापनों के बीच प्राथमिक बाज़ार के प्रदर्शन में गिरावट देखने को मिली है। अतः कथन 2 सही है।

प्रश्न 77. आर्थिक सर्वेक्षण 2022-23 के संदर्भ में निम्नलिखित कथनों पर विचार कीजिये :

  1. थोक मूल्य मुद्रास्फीति (WPI) में  प्राथमिक वस्तु का भार विनिर्मित वस्तु से अधिक होता है।
  2. वित्त वर्ष 2023 में प्राथमिक वस्तुओं और ईंधन तथा बिजली थोक मुद्रास्फीति के मुख्य संचालक थे।
  3. भारतीय रिज़र्व  बैंक के अनुसार अंतर्राष्ट्रीय बाज़ार  में 1% मुद्रास्फीति में वृद्धि भारत के घरेलू बाज़ार  में 10% से अधिक मुद्रास्फीति प्राप्त कर सकती है।

उपर्युक्त कथनों में से कौन सा/से सही नहीं है/हैं?

(A) केवल 1 और 2 
(B) केवल 1 और 3 
(C) 1, 2 और 3
(D) उपरोक्त में से कोई नहीं

उत्तर: B

व्याख्या:

घरेलू थोक मूल्य मुद्रास्फीति: वैश्विक आपूर्ति श्रृंखला विघटन के कारण थोक मूल्य मुद्रास्फीति-

    • कोविड-19 की अवधि में WPI आधारित मुद्रास्फीति कम रही तथा महामारी के बाद की अवधि में आर्थिक गतिविधियों के फिर से शुरू होने पर इसमें तेज़ी आने लगी।
    • WPI में दो अंकीय मुद्रास्फीति को खाद्य मुद्रास्फीति और रूस-यूक्रेन संघर्ष के कारण खराब वैश्विक आपूर्ति श्रृंखलाओं के लिये ज़िम्मेदार ठहराया जा सकता है। अनियमित जलवायु परिस्थितियों के कारण खाद्य मुद्रास्फीति में अनाज और सब्जियों का प्रमुख योगदान था।
    • WPI मुद्रास्फीति का एक भाग आयातित मुद्रास्फीति से है। खाद्य तेलों पर उच्च आयात निर्भरता का अर्थ है कि इन उत्पादों की बढ़ती अंतर्राष्ट्रीय कीमतों का अस्थायी प्रभाव घरेलू कीमतों में भी परिलक्षित होता है।
      • RBI की एक रिपोर्ट (मौद्रिक नीति रिपोर्ट, सितंबर 2022) में प्रदर्शित है कि वैश्विक मुद्रास्फीति में हुए आघात  के कारण सभी देशों और क्षेत्रों में कीमतों में 1% की वृद्धि भारत की मुद्रास्फीति को लगभग 63 आधार अंकों तक बढ़ा सकती है जिसमें घरेलू अप्रत्यक्ष प्रभाव (46 आधार अंक) तथा ग्लोबल स्पिलओवर्स (17 आधार अंक) साथ ही 100 आधार अंक के अप्रत्यक्ष प्रभाव के अतिरिक्त है। अतः कथन 3 सही नहीं है।

    • अतः कथन 2 सही है।

    • अतः कथन 1 सही नहीं है।
    • निवेश कीमतों में मुद्रास्फीति को नियंत्रित करने के उपाय:
      • ईधन की कीमतें: केंद्र सरकार ने पेट्रोल और डीजल पर उत्पाद शुल्क को समायोजित करके हस्तक्षेप किया है।
      • प्लास्टिक उत्पाद: घरेलू विनिर्माण की लागत कम करने के लिये प्लास्टिक उद्योग में इस्तेमाल होने वाले कच्चे माल के आयात पर आयात शुल्क घटा दिया गया है।
      • कपास: सरकार ने कपड़ा उधोग को लाभ पहुंँचाने और उपभोक्ताओं के लिये कीमतों में कमी लाने के लिये कपास के आयात पर सीमा शुल्क माफ कर दिया।
      • हीरे और रत्न: बजट 2022-2023 में तराशे गए हीरों, रत्नों और साधारण से दिखने वाले हीरे पर सीमा शुल्क में कटौती की गई थी।
      • रासायनिक उत्पाद: पेट्रोलियम शोधन के लिये कुछ महत्वपूर्ण रसायनों जैसे मेथनॉल, एसिटिक एसिड और भारी कच्चे माल पर सीमा शुल्क दिया गया।

        प्रश्न 78 आर्थिक सर्वेक्षण 2022-2023 के संदर्भ में निम्नलिखित कथनों पर विचार कीजिये :

        1. थोक मूल्य मुद्रास्फीति (WPI) और उपभोक्ता मूल्य मुद्रास्फीति (CPI) में विचलन था परंतु बाद में इसने रेखाओं की प्रगति की प्रवृत्ति को अपनाया।
        2. आयातित लागत के प्रभाव तथा दो सूचकांकों के सापेक्ष भार में अंतर इन्हीं प्रवृत्तियों के कारण थे।
        3. WPI में हो रही गिरावट की अवधि में CPI ऊपर की ओर बढ़ रही थी।।

        उपर्युक्त कथनों में से कौन सा/से सही नहीं है/हैं?

        (A) केवल 1 और 3
        (B) केवल 2 और 3
        (C) 1, 2 और 3
        (D) उपरोक्त में से कोई नहीं

        उत्तर: D

        व्याख्या:

        • थोक मूल्य मुद्रास्फीति (WPI): WPI ने मई 2022 में 16.6% के शिखर मूल्य के साथ 2022 में बढ़ना शुरू किया (जैसा कि महामारी के बाद आर्थिक गतिविधियाँ फिर से शुरू हुईं और रूस-यूक्रेन संघर्ष ने इसे और अधिक कम कर दिया) तथा यह वर्ष के अंत तक 5.0% तक गिर गई। अतः कथन 1 सही है।
        • WPI और CPI की प्रवृत्तियाँ: 2021 के बाद से  WPI और CPI सूचकांक के बीच एक विचलन रहा है इसके बाद अभिसरण की प्रवृत्ति रही है।
          • विचलन के कारण: दो सूचकांकों के सापेक्ष भार में अंतर और खुदरा कीमतों पर आयातित निवेश लागतों का प्रभाव।
          • रेखाओं की प्रगति के कारण: CPI मुद्रास्फीति में वृद्धि (सेवाओं की लागत में वृद्धि से प्रेरित) के साथ कच्चे तेल, लोहा, एल्यूमीनियम आदि जैसी वस्तुओं की थोक मूल्य सूचकांक मुद्रास्फीति में कमी हुई है।अतः कथन 2 सही है।

        • उपभोक्ता मूल्य मुद्रास्फीति (CPI): किसी राष्ट्र की खुदरा मुद्रास्फीति की गणना करने के लिये उपयोग होने वाले सूचकांक CPI का निर्माण करते हैं, जिसे "बाज़ार टोकरी" के रूप में भी जाना जाता है साथ ही यह घरेलू स्तर पर कीमतों में बदलाव को तय करने के लिये ज़िम्मेदार है।
        • वैश्विक आर्थिक मंदी और ब्याज दर में वृद्धि ने वस्तुओं की कीमतों को कम किया है जिससे थोक मूल्य मुद्रास्फीति में भारी गिरावट हुई है। इस प्रकार भारतीय विनिर्माताओं पर कच्चे माल की कीमत का दबाव कम हुआ है।
          • यहाँ तक कि थोक स्तर पर मुद्रास्फीति कम होने के फलस्वरूप खुदरा कीमतों पर पहले की उच्च निवेश लागतों का प्रभाव पड़ा इसलिये CPI बढ़ रहा था। अतः कथन 3 सही है।
        • आधारभूत मुद्रास्फीति लगभग 6 प्रतिशत पर बनी हुई है तथा इस वर्ष के प्रारंभ में आपूर्ति केआघात के दूसरे दौर के प्रभावों को दर्शाती है।

        प्रश्न 79. आर्थिक सर्वेक्षण 2022-23 के संदर्भ में निम्नलिखित कथनों पर विचार कीजिये :

        1. वित्त वर्ष 2023 में, खुदरा मुद्रास्फीति मुख्य रूप से उच्च खाद्य मुद्रास्फीति से प्रेरित थी जबकि मूल मुद्रास्फीति मध्यम स्तर पर रही।
        2. हेडलाइन मुद्रास्फीति में खाद्य और पेय पदार्थ, कपड़े ,जूते, ईंधन तथा प्रकाश  का सबसे कम योगदान था।

        उपर्युक्त कथनों में से कौन सा/से सही है/हैं?

        (A) केवल 1
        (B) केवल 2
        (C) 1 और 2 दोनों
        (D) न तो 1 और न ही 2

        उत्तर: B

        व्याख्या:

        • अपने चरम से हेडलाइन मुद्रास्फीति में गिरावट: वित्त वर्ष 2022 में वित्त वर्ष 2021 की तुलना में CPI-संयुक्त (CPI-C) आधारित खुदरा मुद्रास्फीति कम देखी गई।
          • वित्त वर्ष 2022 की अवधि में कुछ उप-समूहों जैसे 'तेल और वसा', 'ईंधन और प्रकाश' तथा 'परिवहन और संचार' में उच्च मुद्रास्फीति देखी गई। यह मुख्य रूप से महामारी-प्रेरित लॉकडाउन के कारण आपूर्ति में रुकावटों के कारण था।
          • वित्त वर्ष 2023, रूस-यूक्रेन संकट के साथ शुरू हुआ जिसके कारण अप्रैल 2022 में हेड लाइन मुद्रास्फीति की उच्च दर प्राप्त हुई। वित्त वर्ष 2023 में, खुदरा मुद्रास्फीति मुख्य रूप से उच्च खाद्य मुद्रास्फीति से प्रेरित थी जबकि हेडलाइन मुद्रास्फीति मध्यम स्तर पर रही।

        • खाद्य उत्पादों द्वारा प्रेरित खुदरा मुद्रास्फीति: खुदरा मूल्य मुद्रास्फीति मुख्य रूप से कृषि और संबद्ध क्षेत्र, आवास, कपड़ा और दवा क्षेत्रों से आती है। अतः कथन 1 सही है।
        • वित्त वर्ष 2023 की अवधि में  'खाद्य और पेय पदार्थ 'तथा 'कपड़े और जूते', एवं 'ईंधन और प्रकाश'  हैडलाइन मुद्रास्फीति में प्रमुख योगदानकर्ता थे- पिछले दो वर्षों की तुलना में इस वित्तीय वर्ष में खाद्य और पेय पदार्थ 'तथा 'कपड़े और जूते का योगदान अधिक रहा है। अतः कथन 2 सही नहीं है।
        • कोविड-19 के बाद की अवधि में उपभोक्ता मांग के पुनरुद्धार के कारण 'विविध' समूहों के अंतर्गत, 'घरेलू सामान और सेवाओं' तथा 'व्यक्तिगत देखभाल और प्रभाव' जैसे उपसमूहों में उच्च मुद्रास्फीति दिखाई पड़ती है ।

        प्रश्न 80 समाज के कमज़ोर वर्ग पर मुद्रास्फीति और आपूर्ति की कमी के प्रभाव को कम करने के लिये सरकार द्वारा निम्नलिखित में से कौन-सा/से आवश्यक कदम उठाए गए थे/थे?

        1. प्रधानमंत्री गरीब कल्याण अन्न योजना का शुभारंभ किया।
        2. बफर स्टॉक बनाए रखना और दालों पर आयात शुल्क तथा उपकर में कमी।
        3. विभिन्न देशों द्वारा निर्यात कर की वसूली में वृद्धि । 
        4. 'सोया मील' को एक आवश्यक वस्तु घोषित किया।

        निम्नलिखित कूट का प्रयोग कर सही उत्तर दीजिये:

        (A) केवल 1, 2 और 3
        (B) केवल 2, 3 और 4
        (C) केवल 1, 2 और 4
        (D) 1, 2, 3 और 4  

        उत्तर: C

        व्याख्या:

        वित्त वर्ष 2023 में सब्जियों तथा अनाजों के कारण खाद्य मुद्रास्फीति:

        • उपभोक्ता खाद्य मूल्य सूचकांक (CFPI) पर आधारित खाद्य मुद्रास्फीति वित्त वर्ष 2012 में 3.8 प्रतिशत से बढ़कर वित्त वर्ष 2023 में 7.0 प्रतिशत हो गई। यद्यपि खाद्य मुद्रास्फीति में वृद्धि व्यापक आधार वाली है लेकिन इसमें प्रमुख योगदान सब्जियों, अनाज, दूध और मसालों का है।
        • सितंबर 2022 से अनाजों में दो अंकों की मुद्रास्फीति देखी गई है। गेहूँ और चावल की बढ़ती कीमतों को रोकने के लिये सरकार ने गेहूंँ उत्पादों के निर्यात पर रोक लगा दी तथा चावल पर निर्यात शुल्क लगा दिया है।
        • मूल्य वृद्धि से कमज़ोर वर्गों को बचाने के लिये सरकार ने 80 करोड़ से अधिक लाभार्थियों को मुफ्त खाद्यान्न प्रदान करने के लिये 1 जनवरी 2023 को एक नई एकीकृत खाद्य सुरक्षा योजना 'प्रधानमंत्री गरीब कल्याण अन्न योजना' प्रारंभ की है अतः कथन 1 सही है।
        • उच्च उत्पादन और बफर स्टॉक बनाए रखने तथा दालों पर आयात शुल्क और उपकर को कम करने के संदर्भ में सरकार द्वारा किये गए उपायों के कारण दालों में मुद्रास्फीति स्थिर रही। अतः कथन 2 सही है।
        • वैश्विक उत्पादन में कमी तथा विभिन्न देशों द्वारा निर्यात कर वसूलने में वृद्धि के कारण वित्त वर्ष 2022 में खाद्य तेलों की अंतर्राष्ट्रीय कीमतों में वृद्धि हुई है तथा इसने कीमतों में वृद्धि की न कि उसको घटाया अतः कथन 3 सही नहीं है।
        • आवश्यक खाद्य वस्तुओं में मुद्रास्फीति को नियंत्रित करने के उपाय:
          • अनाज: गेहूंँ के आटे के निर्यात पर प्रतिबंध लगा दिया गया और साथ ही साथ  चावल, ब्राउन राइस तथा सेमी-मिल्ड के साथ ही पूरी तरह से मिल्ड चावल पर 20 प्रतिशत का निर्यात शुल्क लगाया गया था, इनमें उबले हुए चावल को सम्मिलित नही किया गया है ।
          • दालें: मूल्य स्थिरीकरण के लिये दालों का बफर स्टॉक बनाए रखा गया है साथ ही मसूर दाल पर आयात शुल्क एवं कृषि अवसंरचना तथा विकास उपकर (AIDC) को शून्य प्रतिशत पर लाया गया।
          • केंद्र सरकार ने आवश्यक वस्तु अधिनियम-1955 की अनुसूची में संशोधन करके 30 जून 2022 तक 'सोया मील' को आवश्यक वस्तु के रूप में घोषित करने के लिये आवश्यक वस्तु अधिनियम के अंतर्गत एक आदेश अधिसूचित किया है। अतःकथन 4 सही है।

        प्रश्न 81. आर्थिक सर्वेक्षण 2022-23 के संदर्भ में निम्नलिखित कथनों पर विचार कीजिये:

        1. भारत, अपने खाद्य तेलों की 60% मांग को आयात के माध्यम से पूरा करता है तथा सूरजमुखी का तेल हमारे कुल खाद्य तेल आयात का 15% है।
        2. पिछले 5 वर्षों में खाद्य तेल की मात्रा और मूल्य में लगातार वृद्धि हुई है।

        उपर्युक्त  कथनों में से कौन सा/से सही है/हैं?

        (A) केवल 1
        (B) केवल 2
        (C) 1 और 2 दोनों
        (D) न तो 1 और न ही 2

        उत्तर: A 

        व्याख्या:

        • भारत अपने खाद्य तेलों की 60 प्रतिशत मांग को आयात के माध्यम से पूरा करता है, जिससे यह कीमतों में अंतर्राष्ट्रीय उतार-चढ़ाव के लिये कमज़ोर हो जाता है। उदाहरण के लिये सूरजमुखी का तेल हमारे कुल खाद्य तेल आयात का 15 प्रतिशत है तथा यह  मुख्य रूप से यूक्रेन और रूस से आयात किया जाता है। अतः कथन 1 सही है।
        • आयात के माध्यम से देश में उपभोग किये जाने वाले 60% खाद्य तेलों में से ताड़ के तेल का आयात मुख्य रूप से इंडोनेशिया और मलेशिया से आयात किये जाने वाले कुल खाद्य तेल का लगभग 54% है, जबकि सोयाबीन तेल अर्जेंटीना और ब्राज़ील से लगभग 25% आयात किया जाता  है जबकि 15% का आयात  मुख्य रूप से यूक्रेन और रूस से किया जाता है।

        अतः कथन 2 सही नहीं है।

        प्रश्न 82. आर्थिक सर्वेक्षण 2022-23 के संदर्भ में निम्नलिखित कथनों पर विचार कीजिये:

        1. वित्तीय वर्ष 2023 में वित्त वर्ष 2022 की तुलना में ग्रामीण-शहरी मुद्रास्फीति की प्रवृत्ति उलटी है।
        2. वित्त वर्ष 2022-23 के दौरान ग्रामीण ईंधन मुद्रास्फीति शहरी मुद्रास्फीति की तुलना में कम रही।
        3. वित्तीय वर्ष 2022-23 में ग्रामीण तथा शहरी मुद्रास्फीति के बीच व्यापक अंतर देखा गया।

        उपर्युक्त कथनों में से कौन सा/से सही है/हैं?

        (A) केवल 1 और 2
        (B) केवल 2 और 3
        (C) 1, 2 और 3
        (D) उपरोक्त में से कोई नहीं

        उत्तर: A

        व्याख्या:

        ग्रामीण-शहरी मुद्रास्फीति अंतर में गिरावट हुई है:

        • महामारी के वर्षों की अवधि में देखी गई प्रवृत्ति को उलटते हुए चालू वित्त वर्ष की अवधि में ग्रामीण मुद्रास्फीति शहरी मुद्रास्फीति से ऊपर रही। CPI-C आधारित खाद्य मुद्रास्फीति अप्रैल 2022 में वैश्विक खाद्य कीमतों में कमी तथा कृषि लागत में कमी के कारण 8.3% के उच्च स्तर पर पहुँचने के बाद स्थिर हो गई है। अतः कथन 1 सही है।
          • हालाँकि शहरी मुद्रास्फीति के लिये स्थिरता अधिक स्पष्ट थी।
        • चालू वित्त वर्ष की अवधि में ग्रामीण ईंधन मुद्रास्फीति शहरी की तुलना में कम रही है क्योंकि पेट्रोल तथा डीजल के विपरीत जलाऊ लकड़ी और गाय के गोबर के उपले जैसे पारंपरिक ईंधन की कीमतों पर दबाव कम रहा है।
          अतः कथन 2 सही है।
        • जबकि चालू वित्त वर्ष 2022-23 में ग्रामीण तथा शहरी मुद्रास्फीति के बीच व्यापक अंतर देखा गया। अतः कथन 3 सही नहीं है।
          • खाद्य मुद्रास्फीति के अनुभव में अंतर के कारण ग्रामीण तथा शहरी मुद्रास्फीति के बीच का अंतर मार्च 2022 में अपने सबसे व्यापक स्तर पर पहुंँच गया। भीतरी इलाकों की तुलना में शहरी क्षेत्रों में इस अवधि में सब्जियों और तेलों की खाद्य कीमतों में अधिक वृद्धि हुई।
        • राज्यों/केंद्र शासित प्रदेशों में ग्रामीण तथा शहरी मुद्रास्फीति: वित्त वर्ष 2022 की तुलना में वित्त वर्ष 2023 में अधिकांश राज्यों में CPI-C मुद्रास्फीति में वृद्धि हुई है। 
          • तेलंगाना, पश्चिम बंगाल, महाराष्ट्र, मध्यप्रदेश, हरियाणा और आंध्र प्रदेश - मुद्रास्फीति में वृद्धि के लिये ईंधन और कपड़ो के प्रमुख योगदानकर्त्ता थे।
          • हरियाणा, मिज़ोरम और पश्चिम बंगाल के ग्रामीण क्षेत्रों में यह 'खाद्य और पेय पदार्थ' तथा 'कपड़े और जूते में हुई महंगाई के कारण था।
          • मध्य प्रदेश, मणिपुर और असम के ग्रामीण क्षेत्रों में यह 'ईंधन और प्रकाश' के क्षेत्रक के परिणामस्वरूप  था।
          • बिहार, मेघालय और त्रिपुरा के शहरी क्षेत्रों में उच्च मुद्रास्फीति के लिये  भोजन, वस्त्र और ईंधन का प्रमुख योगदान था।

         

        प्रश्न 83. आर्थिक सर्वेक्षण 2022-23 के संदर्भ में निम्नलिखित कथनों पर विचार कीजिये:

        1. भारतीय रिज़र्व  बैंक वित्त वर्ष 2018 को आधार वर्ष मानते हुए दो आवास मूल्य सूचकांक प्रकाशित करता है।
        2. देश के सभी आठ प्रमुख महानगरों ने वार्षिक आधार पर सूचकांक में वृद्धि प्रदर्शित की ।

        उपर्युक्त कथनों में से कौन सा/से सही है/हैं?

        (A) केवल 1
        (B) केवल 2
        (C) 1 और 2 दोनों
        (D) न तो 1 और न ही 2

        उत्तर: B

        व्याख्या:

        आवास मूल्य: महामारी के बाद आवास क्षेत्र में सुधार

        • राष्ट्रीय आवास बैंक (NHB) वित्त वर्ष 2018 को आधार वर्ष मानकर दो आवास मूल्य सूचकांक (HPI): 'HPI मूल्यांकन मूल्य' तथा 'HPI बाज़ार  मूल्य, ‘त्रैमासिक' आधार पर प्रकाशित करता है। अतः कथन 1 सही नहीं है।
          • HPI मूल्यांकन मूल्य प्राथमिक ऋण देने वाली संस्थाओं से एकत्रित आवासीय इकाइयों के मूल्यांकन मूल्यों पर आधारित है।
          • इसके विपरीत HPI बाज़ार मूल्य डेवलपर्स से एकत्र की गई तथा बेची गई सूची के बाज़ार मूल्य पर आधारित है। भार के रूप में शहरों की जनसंख्या का उपयोग करके संपूर्ण भारत के 50 शहरों के लिये  एक समग्र सूचकांक की गणना की जाती है।
        • 50 शहरों में से 43 में सूचकांक में वृद्धि देखी गई जबकि 7 शहरों में वार्षिक गिरावट देखी गई। देश के सभी आठ प्रमुख महानगरों जैसे अहमदाबाद, बेंगलुरु, चेन्नई, दिल्ली, हैदराबाद, कोलकाता, मुंबई और पुणे ने वार्षिक आधार पर सूचकांक में वृद्धि दर्ज की। अतः कथन 2 सही है।
        • कोविड-19 संकट ने आवासीय अचल संपत्ति बाज़ार को काफी प्रभावित किया। अप्रैल-जून, 2020 में महामारी के प्रारंभिक चरण में स्वास्थ्य संबंधी चिंताओं और घर पर रहने के आदेशों के कारण घरों के खरीदारों की संख्या कम हो गई थी।
          • हालाँकि कम आवास ऋण की ब्याज दरों के साथ सरकार द्वारा समय पर नीतिगत हस्तक्षेप ने मांग को बढ़ाया तथा वित्त वर्ष 2023 में किफायती खंड में खरीदारों को अधिक आसानी से आकर्षित किया। समग्र HPI मूल्यांकन और HPI बाज़ार कीमतों में समग्र वृद्धि आवास वित्त क्षेत्र में पुनरुद्धार का संकेत देती है। HPI में स्थिर से मध्यम वृद्धि भी संपत्ति के बरकरार मूल्य के संदर्भ में घर के मालिकों और आवास ऋण फाइनेंसरों को विश्वास प्रदान करती है।

        आवास वित्त क्षेत्र (HFC) को NHBs की सहायता:

        • आज़ादी का अमृत महोत्सव (AKAM) के एक भाग के रूप में NHB ने महिलाओं, ग्रामीण क्षेत्रों, SC/ST, आकांक्षी ज़िलों, उत्तर-पूर्वी क्षेत्र, ऋण सहित विभिन्न श्रेणियों के लिये पुनर्वित्त के तहत 25/30 आधार अंकों की छूट दी हैं।
        • लिंग /निःशक्तजन / विकलांग, जम्मू और कश्मीर तथा लद्दाख के केंद्र शासित प्रदेश और ग्रीन हाउसिंग।
        • 2070 तक निवल-शून्य उत्सर्जन तक पहुँचने के लिये कोप-26 में राष्ट्र की प्रतिबद्धता को देखते हुए NHBs द्वारा ग्रीन हाउसिंग के तहत ऋण के लिये 100 bps की छूट दी है।

        प्रश्न 84. निम्नलिखित कथनों पर विचार कीजिये :

        1. एक अंतर्राष्ट्रीय मुद्रा वह मुद्रा है जो जारी करने वाले देश की सीमाओं से परे भी उपयोग की जाती है। यह मुद्रा न केवल उस देश निवासियों, बल्कि गैर-निवासियों के मध्य भी लेनदेन के लिये  महत्त्वपूर्ण होती है।
        2. हाल ही में, RBI ने अंतर्राष्ट्रीय मुद्रा के रूप में भारतीय रुपए (INR) में वैश्विक व्यापारिक समुदाय की रुचि बढ़ाने के लिये भारतीय रुपए (INR) में इनवॉइस, भुगतान और निर्यात/आयात के समझोते के लिये एक अतिरिक्त व्यवस्था की अनुमति दी है।

        उपर्युक्त कथनों में से कौन-सा/से सही है/हैं?

        (A) केवल 1
        (B) केवल 2
        (C) 1 और 2 दोनों
        (D) न तो 1 न ही 2

        उत्तर: c 

        व्याख्या

        • अंतर्राष्ट्रीय मुद्रा वह मुद्रा है ,जो जारी करने वाले देश की सीमाओं से परे उपयोग की जाती है  जिसे न केवल उस देश के निवासियों, बल्कि गैर-निवासियों के मध्य भी लेनदेन के लिये महत्त्वपूर्ण माना जाता है।
        • दूसरे शब्दों में, एक अंतर्राष्ट्रीय मुद्रा वह मुद्रा है जिसका उपयोग अंतर्राष्ट्रीय लेनदेन में प्रत्यक्ष रूप से शामिल पक्षकारों की राष्ट्रीय मुद्राओं के बदले में किया जाता है, फिर लेनदेन में चाहे वस्तुओं और सेवाओं की खरीद शामिल हो  या वित्तीय संपत्तियों की की लेनदेन शामिल हो। अतः कथन 1 सही है।
        • भारत सरकार ने भारतीय रुपए (INR) में अंतर्राष्ट्रीय व्यापार समझोते की अनुमति देने के लिये विदेश व्यापार नीति में और प्रक्रियाओं की पुस्तिका में आवश्यक संशोधन किये हैं, यानी भारतीय रुपए (INR) में इनवॉइस, भुगतान और निर्यात/आयात के समझोते के लिये एक अतिरिक्त व्यवस्था की अनुमति दी है। यह RBI के दिशा-निर्देशों के अनुसार किया गया है।
        • भारतीय रुपये के अंतर्राष्ट्रीयकरण में वैश्विक समुदाय की रुचि में वृद्धि को देखते हुए और  भारतीय रुपये में अंतर्राष्ट्रीय व्यापार को सुविधाजनक बनाने हेतु नीतिगत संशोधन किये गए हैं। अतः कथन 2 सही है।

        प्रश्न 85. वित्तीय वर्ष 2021-22 के संदर्भ में निम्नलिखित कथनों पर विचार कीजिये:

        1. भारत के कुल आयात में चीन, UAE, अमेरिका, रूस और सऊदी अरब की संयुक्त हिस्सेदारी 40 प्रतिशत  है।
        2. संयुक्त अरब अमीरात और नीदरलैंड के उपरांत अमेरिका निर्यात का शीर्ष गंतव्य रहा है ।
        3. नीदरलैंड ने चीन को, भारत के निर्यात भागीदार के रूप में तीसरे स्थान से स्थानांतरित किया है।

        उपर्युक्त कथनों में से कौन-सा/से सही है/हैं?

        (A) केवल 1
        (B) केवल 1 और 2 
        (C) केवल 2 और 3
        (D) उपरोक्त सभी

        उत्तर: D

        व्याख्या :

        • आयात के संबंध में, चीन, संयुक्त अरब अमीरात, अमेरिका, रूस और सऊदी अरब  भारत के कुल आयात में संयुक्त रूप से 40 प्रतिशत की हिस्सेदारी रखते है। अतः कथन 1 सही है।
          • हालाँकि, अप्रैल से नवंबर 2022 के दौरान चीन की हिस्सेदारी एक वर्ष पहले की तुलना में  15.5% से घटकर 13.8% हो गई है। इसी तरह अप्रैल से नवंबर 2022 में अमेरिका की हिस्सेदारी एक वर्ष पहले की तुलना  में 7.2% से घटकर 6.9% रह गई।
        • अप्रैल-नवंबर 2022 में संयुक्त अरब अमीरात और नीदरलैंड के उपरांत, अमेरिका निर्यात का शीर्ष गंतव्य रहा। अतः कथन 2 सही है।
        • नीदरलैंड ने चीन को, भारत के निर्यात भागीदार के रूप में तीसरे स्थान से स्थानांतरित किया है। अतः कथन 3 सही है।
        • भारत  समय के साथ अपने निर्यात स्थलों में विविधता लाया है। उदाहरण के लिये, कुल निर्यात में दक्षिण अफ्रीका की हिस्सेदारी FY19 में 1.2% से बढ़कर FY23 (अप्रैल से नवंबर) में 2.0 प्रतिशत हो गई है।

        प्रश्न 86. भारत के विदेशी क्षेत्र के लचीलेपन में सुधार हेतु क्या उपाय किये जा रहे हैं?

        (A) मुक्त व्यापार समझौते
        (B) भारतीय रुपए में अंतर्राष्ट्रीय समझौते 
        (C) राष्ट्रीय रसद नीति
        (D) उपरोक्त सभी

        उत्तर D

        व्याख्या:

        • भारत के विदेशी क्षेत्र के लचीलेपन में सुधार लाने के लिये जिन उपायों पर विचार/कार्यान्वयन किया जा रहा है, इनमें  मुक्त व्यापार समझौते, राष्ट्रीय रसद नीति और भारतीय रुपए में अंतर्राष्ट्रीय व्यापार समझौते को बढ़ावा देने के प्रयास शामिल हैं।
        • मुक्त व्यापार समझौते रियायती शुल्क और गैर-शुल्क बाधाओं पर निर्यात के अवसर पैदा करके बाह्य बाधाओं को संबोधित करेंगे।
        • भारतीय रूपये में अंतर्रार्ष्‍टीय व्‍यापार समझौते को बढ़ावा देने के प्रयास किये जा रहे है। एक बार इन पहलों से लाभ प्राप्‍त होने पर विदेशी मुद्रा पर निर्भरता व्‍यापक रूप से कम होगी, जिससे अर्थव्‍यवस्‍था को विदेशी आघातों से कम संवेदी बनाया जा सकेगा।
        • राष्ट्रीय रसद नीति, आंतरिक रसद की लागत को कम करके भारतीय निर्यात को प्रोत्साहित करने के लिये घरेलू  बाधाओं को कम करेगी।
        • अतः विकल्प D सही है।

        प्रश्न 87. निम्नलिखित कथनों पर विचार कीजिये : 

        1. भारत में सबसे बड़ी प्रवासी आबादी है, यह शीर्ष प्रेषण प्राप्तकर्त्ता देश है।
        2. प्रेषण बाह्य वित्तपोषण का सबसे बड़ा प्रमुख स्रोत हैं।
        3. खाड़ी सहयोग परिषद (GCC) देशों में कार्यरत कौशलहीन, अनौपचारिक रूप से रोज़गार प्राप्त भारतीय प्रवासियों के  यूनाइटेड किंगडम, पूर्वी एशिया तथा संयुक्त राज्य अमेरिका जैसे उच्च-आय वाले देशों ( जहाँ उच्च-कुशल नौकरियों का एक प्रमुख हिस्सा है) में स्थानांतरण से इनके प्रमुख गंतव्यों में एक क्रमिक संरचनात्मक बदलाव हुआ और प्रेषण लाभ में वृद्धि हुई है।

        उपर्युक्त कथनों में से कौन-सा/से सही है/हैं?

        (A) केवल 1
        (B) केवल 1 और 2 
        (C) केवल 1 और 3
        (D) उपरोक्त सभी

        उत्तर C 

        व्याख्या :

        • भारत में सबसे बड़ी प्रवासी आबादी है, विश्व बैंक के अनुसार वर्ष 2022 में यह 100 बिलियन अमेरिकी डॉलर तक पहुँचने वाला प्रत्याशित प्रेषण के साथ शीर्ष प्रेषण प्राप्तकर्ता देश है। अतः कथन 1 सही है।
        • सेवा निर्यात के बाद प्रेषण बाह्य वित्तपोषण का दूसरा सबसे बड़ा प्रमुख स्रोत है, जो CAD (चालू खाता घाटा) को न्यूनतम करने में योगदान प्रदान करता है, यह सदैव BoP (भुगतान संतुलन) का एक स्थिर घटक रहा है। अतः कथन 2 सही नहीं है।
        • खाड़ी सहयोग परिषद (GCC) देशों में कार्यरत कौशलहीन, अनौपचारिक रूप से रोज़गार प्राप्त भारतीय प्रवासियों के  यूनाइटेड किंगडम, पूर्वी एशिया तथा संयुक्त राज्य अमेरिका जैसे उच्च-आय वाले देशों ( जहाँ उच्च-कुशल नौकरियों का एक प्रमुख हिस्सा है) में स्थानांतरण से इनके प्रमुख गंतव्यों में एक क्रमिक संरचनात्मक बदलाव हुआ और प्रेषण लाभ में वृद्धि हुई है। अतः कथन 3 सही है।

        प्रश्न 88. 'अंतर्राष्ट्रीय निवेश स्थिति (IIP)' के संदर्भ में निम्नलिखित कथनों पर विचार कीजिये :

        1. भारत की शुद्ध अंतर्राष्ट्रीय निवेश स्थिति ऋणात्मक है।
        2. कुल अंतर्राष्ट्रीय निवेश स्थिति यह निर्धारित करती है कि कोई देश अपनी बाह्य संपत्ति और देनदारियों में अंतर को माप कर एक शुद्ध लेनदार या ऋणी राष्ट्र है अथवा नहीं।

        उपर्युक्त कथनों में से कौन-सा/से सही है/हैं?

        (A) केवल 1
        (B) केवल 2
        (C) 1 और 2 दोनों
        (D) न तो 1 न ही 2

        उत्तर : C

        व्याख्या :

        • भारत की अंतर्राष्ट्रीय निवेश स्थिति ऋणात्मक है। सितंबर 2022 के अंत तक भारत की अंतर्राष्ट्रीय वित्तीय संपत्तियाँ अंतर्राष्ट्रीय वित्तीय देनदारियों का 68.5% भाग पूर्ण करती थीं। अतः कथन 1 सही है।
        • अंतर्राष्ट्रीय निवेश स्थिति (IIP) एक सांख्यिकीय विवरण है जो एक समय में मूल्य और संरचना को दर्शाती है (a) एक अर्थव्यवस्था के निवासियों की वित्तीय संपत्ति जो कि गैर निवासियों और आरक्षित संपत्ति के रूप में रखे गए स्वर्ण बुलियन (सोने के उत्पाद) पर दावा करती है, और (b) गैर निवासियों के लिये एक अर्थव्यवस्था के निवासियों की देनदारियाँ।
        • अर्थव्यवस्था की बाह्य वित्तीय संपत्तियों और देनदारियों के बीच का अंतर अर्थव्यवस्था का शुद्ध IIP होता है, जो सकारात्मक अथवा नकारात्मक हो सकता है। अतः कथन 2 सही है।
        • शुद्ध IIP यह निर्धारित करता है कि कोई देश अपनी बाह्य संपत्ति और देनदारियों में अंतर को माप कर एक शुद्ध लेनदार या ऋणी राष्ट्र है अथवा नहीं।
        • ये आँकड़े किसी देश की वित्तीय स्थिति और सुदृढ़ता के संकेतक के रूप में कार्य करते हैं। शुद्ध IIP, BoP, घरेलू अर्थव्यवस्था के अंतर्राष्ट्रीय लेखों के समूह को दर्शाता है।

        प्रश्न 89. वित्तीय वर्ष 2021-22 में भारत के भुगतान संतुलन (BoP) के संदर्भ में निम्नलिखित कथनों पर विचार कीजिये :

        1. CAD (चालू खाता घाटा) के बढ़ने के पीछे का कारण तेल की कीमतों में तीव्र वृद्धि है।
        2. भारत विश्व का सबसे बड़ा विदेशी मुद्रा भंडार धारक है।

        उपर्युक्त कथनों में से कौन-सा/से सही है/हैं? 

        (A) केवल 1
        (B) केवल 2
        (C) 1 और 2 दोनों
        (D) न तो 1 न ही 2

        उत्तर : A

        व्याख्या :

        • प्रतिकूल वैश्विक आर्थिक स्थिति ने वर्ष 2022 में भारत के भुगतान संतुलन (BoP) पर दबाब डालने का कार्य किया है।
        • तेल की कीमतों में तीव्र वृद्धि का प्रभाव CAD(चालू खाता घाटा) के विस्तार, फेडरल रिज़र्व (अमेरिकी केंद्रीय बैंक) द्वारा सख्त नीति और अमेरिकी डॉलर के मज़बूत होने के कारण FPI के बहिर्वाह (विदेशी शेयर समूह में निवेश) से स्पष्ट था। अतः कथन 1 सही है।
        • सितंबर 2022 के अंत तक भारत का विदेशी मुद्रा भंडार 532.7 बिलियन अमेरिकी डॉलर था, जिसमें 8.8 माह का आयात शामिल था। दिसंबर 2022 के अंत तक 9.3 माह के आयात को पूर्ण करते हुए यह भंडार बढ़कर 562.7 बिलियन अमरीकी डॉलर हो गया।
        • IMF (अंतर्राष्ट्रीय मुद्रा कोष) द्वारा संकलित आँकड़ों के अनुसार, नवंबर 2022 के अंत तक, भारत विश्व का छठा सबसे बड़ा विदेशी मुद्रा भंडार धारक था। अतः कथन 2 सही नहीं है।

        प्रश्न 90. निम्नलिखित कथनों पर विचार कीजिये :

        1. कोविड-19 महामारी के कारण, वित्तीय वर्ष 2022 में भारत ने विश्व सेवा व्यापार में अपना प्रभुत्व खो दिया।
        2. सॉफ्टवेयर और व्यावसायिक सेवाएँ मिलकर भारत के कुल सेवाओं के निर्यात का 60% से अधिक हिस्सा निर्मित करती हैं।

        उपर्युक्त कथनों में से कौन-सा/से सही है/हैं?

        (A) केवल 1
        (B) केवल 2
        (C) 1 और 2 दोनों
        (D) न तो 1 न ही 2

        उत्तर : B

        व्याख्या :

        • भारत ने वित्तीय वर्ष 2022 में विश्व सेवा व्यापार में अपना प्रभुत्व बनाए रखा है। महामारी के कारण वैश्विक प्रतिबंधों और कमज़ोर पर्यटन राजस्व के बावजूद, वित्तीय वर्ष 22 में भारत का सेवा निर्यात 254.5 बिलियन अमेरिकी डॉलर रहा, जिसमें वित्तीय वर्ष 21 की तुलना में 23.5% की वृद्धि दर्ज हुई है, अप्रैल-सितंबर 2022 में वित्तीय वर्ष 2022 की समान अवधि में 32.7% की वृद्धि दर्ज़ की गई। अतः कथन 1 सही नहीं है।
        • सॉफ्टवेयर और व्यावसायिक सेवाएँ मिलकर भारत के कुल सेवाओं के निर्यात का 60% से अधिक हिस्सा निर्मित करती हैं, वित्तीय वर्ष 2023 में मज़बूत वृद्धि देखने को मिली है।
        • सूचना प्रौद्योगिकी (IT) कंपनियों में खुदरा और उपभोक्ता व्यवसाय जैसे विभिन्न क्षेत्रों से मज़बूत राजस्व; संचार और मीडिया; स्वास्थ्य देखभाल; बैंकिंग, वित्तीय एवं बीमा सेवाओं ने सॉफ्टवेयर निर्यात में वृद्धि को गति प्रदान की, इंजीनियरिंग में एक बेहतरीन तीव्रता, अनुसंधान एवं विकास संबंधी सेवाओं ने तिमाही के दौरान व्यापारिक सेवाओं के निर्यात में वृद्धि को बढ़ावा दिया। अतः कथन 2 सही है।

        प्रश्न 91.  निम्नलिखित कथनों पर विचार कीजिये :

        1. भारत के विदेशी ऋण के लिये इष्टतम सीमा सकल घरेलू उत्पाद का लगभग 23-24% है।
        2. वित्तीय वर्ष 1999  के बाद से SED (संप्रभु विदेशी ऋण) की तुलना में गैर-संप्रभु विदेशी ऋण में वृद्धि देखी गई है।

        उपर्युक्त कथनों में से कौन-सा/से सही है/हैं?

        (A) केवल 1
        (B) केवल 2
        (C) 1 और 2 दोनों
        (D) न तो 1 न ही 2

        उत्तर : C

        व्याख्या :

        • सितंबर 2022 के अंत तक भारत का विदेशी ऋण 610.5 बिलियन अमेरिकी डॉलर था, जो सितंबर 2021 के अंत तक 602.9 बिलियन अमेरिकी डॉलर से 1.3% अधिक था। हालाँकि यह एक वर्ष पूर्व 20.3% था।
        • आर्थिक सर्वेक्षण में भारत के विदेशी ऋण के लिये इष्टतम सीमा सकल घरेलू उत्पाद का लगभग 23-24% है। इस प्रकार, जहाँ तक विदेशी ऋण का संबंध है, भारत को संभावित विकास का सकारात्मक स्थान हासिल है। अतः कथन 1 सही है।
        • वित्त मंत्रालय की "भारत का विदेशी ऋण: स्थिति रिपोर्ट 2019-20", भारत के विदेशी ऋण के विकास का विवरण प्रदान करती है। वित्तीय वर्ष 1999 के बाद से SED (संप्रभु विदेशी ऋण) की तुलना में गैर-संप्रभु विदेशी ऋण में वृद्धि का प्रत्यक्ष रूप से रुझान देखा गया है। तद्नुसार, एक सामान्य वर्ष में यह गैर-संप्रभु विदेशी ऋण सापेक्ष है जो देश के विदेशी ऋण की गतिशीलता को प्रभावित करती है। अतः कथन 2 सही है।
        • वित्तीय वर्ष 2021 के महामारी वर्ष में, SED में वृद्धि हुई जो बहुपक्षीय संस्थानों से कोविड -19 के कारण विदेशी ऋण के समग्र विकास के एक बड़े हिस्से के लिये ज़िम्मेदार थी।
        • जैसे-जैसे महामारी में कमी आई और अर्थव्यवस्था के पुनरुद्धार के साथ सामान्य स्थिति बहाल हुई, भारत के विदेशी ऋण की सामान्य गतिशीलता गैर-SED में वृद्धि के रूप में वापस आ गई, जो मार्च 2022 के अंत में 8.0% की कुल विदेशी ऋण वृद्धि से 4.7 प्रतिशत अधिक थी।
        • हालाँकि, सितंबर 2022 को समाप्त तिमाही में फिर से सामान्य गतिशीलता में विचलन देखा गया।यह SED में सापेक्ष वृद्धि थी जिसने विदेशी सहायता के साथ-साथ अन्य सरकारी ऋण में समग्र विदेशी ऋण गतिशीलता को प्रभावित किया जिसमें G-Sec और SDR में FPI निवेश शामिल है।
        • भारत के बाह्य ऋण में 0.4% की समग्र गिरावट में से, SED में गिरावट 0.5% थी, जबकि गैर-SED में वृद्धि 0.2% थी।
        close
        एसएमएस अलर्ट
        Share Page
        images-2
        images-2